You are on page 1of 67

Total Marks : 200

Online Prelims TEST - 34 (TEXTBOOK)


( InsightsIAS Mock Test Series for UPSC Preliminary Exam 2020 ) Mark Scored : 0

1 Wetlands provide a wide range of important ecosystem services, which include


1. Water purification
2. Flood moderation
3. Erosion control
4. Microclimate regulation

Select the correct answer using the codes below.


A. 2 and 3 only
B. 1, 2 and 3 only
C. 1 and 4 only
D. 1, 2, 3 and 4

Correct Answer : D

Answer Justification :

Justification: Wetlands are particularly important providers of all water-related ecosystem


services. They regulate water quantity, groundwater recharge, and can contribute to regulating
floods and the impacts of storms.

For e.g. Wetlands help in erosion control and sediment transport, thereby contributing to land
formation and increasing resilience to storms. All these ecosystem services improve water security,
including security from natural hazards and climate change adaptation. The final Rio+20 outcome
document, “The Future We Want”, inter alia, recognised the role of ecosystems in the supply of
water and its quality.

Wetlands are productive areas for plant life, animals and wetland agriculture. Compared to many
other ecosystems, wetlands are one of the most productive habitats in the world.

With greater species diversity nutrient recycling and niche specialization than most other
ecosystems. Wetlands are the major habitat for most of the world’s waterbirds and key habitat for
migratory species. Almost all of the world’s waterbirds use wetlands as feeding and breeding
grounds. Migratory waterbirds use wetlands throughout their range which can sometimes literally
be from pole to pole. The feeding, breeding and stop-over areas across and between continents that
migratory birds depend on requires coordinated wetlands conservation efforts among many nations.

Wetlands are an important source of food. Well-managed rice paddy systems, for example, produce
not only rice but also co-benefits from rice-associated biodiversity, such as highly nutritious food in
the form of fish, molluscs and crustaceans.

Q Source: https://www.cbd.int/waters/doc/wwd2015/wwd-2015-press-briefs-en.pdf

2 Which of these ragas is usually performed late in the morning?

A. Raga Darbari
B. Todi
C. Raga Bhopali

1
Total Marks : 200
Online Prelims TEST - 34 (TEXTBOOK)
( InsightsIAS Mock Test Series for UPSC Preliminary Exam 2020 ) Mark Scored : 0

D. Bhimpalasi

Correct Answer : B

Answer Justification :

Justification: Todi should be performed in the late morning. It is a Hindustani classical raga which
gave its name to the Todi thaat, one of the ten types of classical music according to the musicologist
Bhatkhande. Todi is nearly always shown as a gentle, beautiful woman, holding a veena and
standing in a lovely green forest, surrounded by deer.

Raga Bhopali should be sung after sunset.

Bhimpalasi should be sung in late Afternoon.

Raga Darbari is a raga in the Kanada family, which is thought to have originated in Carnatic music
and brought into North Indian music by Miyan Tansen, the 16th-century composer in emperor
Akbar's court.

Q Source: Based on past year UPSC papers

3 The office of the Secretary of state for India was abolished and his functions were transferred to the
secretary of the state for Commonwealth Affairs by which of the following acts?

A. Act of 1858
B. Act of 1919
C. Government of India Act, 1935
D. Indian Independence Act, 1947

Correct Answer : D

Answer Justification :

Learning: The Act abolished the office of viceroy and provided, for each dominion, a governor-
general, who was to be appointed by the British King on the advice of the dominion cabinet.

His Majesty’s Government in Britain was to have no responsibility with respect to the Government
of India or Pakistan.

It also abolished the office of the secretary of state for India and transferred his functions to the
secretary of state for Commonwealth Affairs.

Q Source: Chapter 1: Indian Polity: M Laxmikanth

4 The objective(s) of the Market Intervention Scheme (MIS) scheme is/are


1. To incentivize bumper production of crops for export promotion

2
Total Marks : 200
Online Prelims TEST - 34 (TEXTBOOK)
( InsightsIAS Mock Test Series for UPSC Preliminary Exam 2020 ) Mark Scored : 0

2. To protect the growers of certain horticultural/agricultural commodities from distress sale

Select the correct answer using the codes below.


A. 1 only
B. 2 only
C. Both 1 and 2
D. None of the above

Correct Answer : B

Answer Justification :

Justification: The Department of Agriculture & Cooperation is implementing a scheme namely


Market Intervention Scheme (MIS) for procurement of agricultural and horticultural commodities
which are generally perishable in nature. The basic objective of MIS is to provide remunerative
prices to the farmers in case of glut in production and fall in prices.

Further, the MIS is implemented on the specific requests of the State Government/UT
Administrations willing to share the loss with Central Government on 50:50 basis (75:25 in case of
North-Eastern States). Under MIS, funds are not allocated to the States. Instead, central share of
losses as per the guidelines of MIS is released to the State Governments/UTs, for which MIS has
been approved based on specific proposals received from them.

The Scheme is implemented when there is at least 10% increase in production or 10% decrease in
the ruling rates over the previous normal year.

Under MIS, funds are not allocated to the States. Instead, central share of losses as per the
guidelines of MIP is released to the State Governments/UTs, for which MIP has been approved,
based on specific proposals received from them. The area of operation is restricted to the concerned
state only.

The MIS has been implemented in case of commodities like apples, kinnoo/malta, garlic, oranges,
galgal, grapes, mushrooms, clove, black pepper, pineapple, ginger, red-chillies, coriander seed etc.

Q Source: http://agricoop.nic.in/sites/default/files/MIS_0.pdf

5 Consider the following statements.


1. Ramosis Rising of 1820s and 1840s occurred over water rights in the region of Western Ghats.
2. Cutch Rebellion in 1819 occurred because the British took control of the salt production at the
coasts.

Which of the above is/are correct?


A. 1 only
B. 2 only
C. Both 1 and 2
D. None

3
Total Marks : 200
Online Prelims TEST - 34 (TEXTBOOK)
( InsightsIAS Mock Test Series for UPSC Preliminary Exam 2020 ) Mark Scored : 0

Correct Answer : D

Answer Justification :

Learning: The British interfered in the internal feuds of the Cutch and, in 1819, defeated and
deposed the Cutch ruler Rao Bharamal in favour of his infant.

A British resident governed the areas as the de facto ruler with the help of a regency council.

The administrative innovations made by the regency council coupled with excessive land
assessment caused deep resentment.

The Ramosis, the hill tribes in the Western Chats, resented British rule and the British pattern of
administration. In 1822, under Chittur Singh, they revolted and plundered the country around
Satara. There were revolts again during 1825-1826 and the area remained disturbed till 1829. The
disturbance erupted again in 1840-1841 over deposition and banishment of Raja Pratap Singh of
Satara in September 1839. A superior British force restored order in the area.

Q Source: Improvisation: Chapter 2: Bipin Chandra: India’s Struggle for Independence

6 Consider the following with regards to recently released RBI guidelines for on-tap licencing of private
sector SFBs.
1. Payments banks can apply for conversion into small finance banks (SFBs).
2. Cooperative banks cannot be converted into SFBs.
3. The promoter of a payments bank cannot set up SFB as long as his payments bank is operational.

Select the correct answer using the codes below.


A. 1 and 2 only
B. 1 only
C. 2 and 3 only
D. 1, 2 and 3

Correct Answer : B

Answer Justification :

Justification: RBI has announced the final guidelines for on-tap licencing of private sector SFBs.
These guidelines include:

Payments banks can apply for conversion into small finance banks (SFBs) after five years of
operation.
The promoter of a payments bank is eligible to set up an SFB, provided that both banks come
under the non-operating financial holding company (NOFHC) structure.
The minimum paidup capital requirement for SFBs has been raised from ₹100 crore to ₹200
crore.
SFBs should be listed within three years of reaching a net worth of ₹500 crore. They will be

4
Total Marks : 200
Online Prelims TEST - 34 (TEXTBOOK)
( InsightsIAS Mock Test Series for UPSC Preliminary Exam 2020 ) Mark Scored : 0

given scheduled bank status immediately upon commencement of operations, and will have
general permission to open banking outlets from the date of commencement of operations

Statement 2: Primary urban cooperative banks can convert into SFBs, provided they comply with
the ontap licencing guidelines. The minimum net worth of such SFBs will be ₹100 crore and has to
be increased to ₹200 crore within five years from commencement of business.

Learning: Existing rules do not allow payments banks to lend and deposits are capped at ₹1 lakh
per customer. If these entities get the licence of small finance banks, it will give them access to
more deposits and boost their profitability, which is at present under pressure.

Q Source: RBI guidelines: Website

7 Consider the following about the “Adaptation fund”.


1. It was established under the Kyoto Protocol of the UN Framework Convention on Climate Change.
2. It finances projects and programmes that help vulnerable communities in developing countries
adapt to climate change.

Select the correct answer using the codes below.


A. 1 only
B. 2 only
C. Both 1 and 2
D. None of the above

Correct Answer : C

Answer Justification :

Justification: Established under the Kyoto Protocol of the UN Framework Convention on Climate
Change – it finances projects and programmes that help vulnerable communities in developing
countries adapt to climate change. Initiatives are based on country needs, views and priorities.

The Fund is financed in part by government and private donors, and also from a two percent share
of proceeds of Certified Emission Reductions (CERs) issued under the Protocol’s Clean Development
Mechanism projects.

The Fund is supervised and managed by the Adaptation Fund Board (AFB). The AFB is composed of
16 members and 16 alternates and meets at least twice a year.

The World Bank serves as trustee of the Adaptation Fund on an interim basis.

Q Source: Insights current events: Environment Compilation

8 Consider the following statements.


1. Indian crocodile, or Mugger, is primarily a saltwater species.
2. The Gharial or fish-eating crocodile is native to the Indian subcontinent.

5
Total Marks : 200
Online Prelims TEST - 34 (TEXTBOOK)
( InsightsIAS Mock Test Series for UPSC Preliminary Exam 2020 ) Mark Scored : 0

Select the correct answer using the codes below.


A. 1 only
B. 2 only
C. Both 1 and 2
D. None of the above

Correct Answer : B

Answer Justification :

Justification: There are three species of crocodilians—saltwater, Mugger and Gharial.

1. Mugger: o The mugger crocodile, also called the Indian crocodile, or marsh crocodile, is found
throughout the Indian subcontinent. o It is listed as vulnerable by IUCN. o The mugger is mainly a
freshwater species, and found in lakes, rivers and marshes.

2. Gharial: o The Gharial or fish-eating crocodile is native to the Indian subcontinent. o It is listed as
a Critically Endangered by IUCN. o Small released populations are present and increasing in the
rivers of the National Chambal Sanctuary, Katarniaghat Wildlife Sanctuary, Son River Sanctuary
and the rainforest biome of Mahanadi in Satkosia Gorge Sanctuary, Orissa.

3. Saltwater Crocodile: o It is the largest of all living reptiles. It listed as least concern by IUCN. It
is found throughout the east coast of India.

Bhitarkanika census finds an increase of 15 saltwater crocodiles from last year. There are now
1,757 crocodiles in the park, the census conducted on January 3, 2020, found. Last year, there were
1,742.

Q Source: As mentioned above

9 GOCO model, recently in news, concerns with

A. Addressing refugee crisis in Myanmar


B. Re-balancing defence expenditure
C. Building land boundaries in Bangladeshi enclaves
D. Hiking R&D expenditure in pandemic prevention

Correct Answer : B

Answer Justification :

Justification: The GOCO model was one of the recommendations of the Lt. Gen. DB Shekatkar
(Retd.) committee to “enhance combat capability and re-balancing defence expenditure.”

How it works?

6
Total Marks : 200
Online Prelims TEST - 34 (TEXTBOOK)
( InsightsIAS Mock Test Series for UPSC Preliminary Exam 2020 ) Mark Scored : 0

The assets owned by government will be operated by the private industries.


Under the model, the private companies need not make investments on land, machinery and
other support systems.
The missions are set by government and the private sectors are given full independence in
implementing the missions using their best practices.

Q Source: Insights: Economy compilation

10 A geostationary orbit is a particular type of geosynchronous orbit where a satellite in geosynchronous


orbit returns to the same point in the sky at the same time each day. This is made possible as
1. The satellite orbits directly above the poles.
2. The satellite follows the direction of the Earth's rotation.

Which of the above is/are correct?


A. 1 only
B. 2 only
C. Both 1 and 2
D. None

Correct Answer : B

Answer Justification :

Justification: It is a circular orbit nearly 36,000 kilometres above the Earth's equator following the
direction of the Earth's rotation.

An object in such an orbit has an orbital period equal to the Earth's rotational period (one sidereal
day), and thus appears motionless, at a fixed position in the sky, to ground observers.

Communications satellites and weather satellites are often placed in geostationary orbits, so that
the satellite antennas (located on earth) which communicate with them do not have to rotate to
track them, but can be pointed permanently at the position in the sky where the satellites are
located

Q Source: Based on UPSC past year papers

11 Consider the following statements.


1. Open Market Operations (OMOs) include both outright purchase and sale of government securities
for adjusting liquidity.
2. Market Stabilisation Scheme (MSS) allows large capital inflows to be absorbed through sale of
short-dated government securities and treasury bills.

Select the correct answer using the codes below.


A. 1 only
B. 2 only
C. Both 1 and 2

7
Total Marks : 200
Online Prelims TEST - 34 (TEXTBOOK)
( InsightsIAS Mock Test Series for UPSC Preliminary Exam 2020 ) Mark Scored : 0

D. None of the above

Correct Answer : C

Answer Justification :

Justification: We have covered them in some detail in previous test as well. This is for revision.

Open Market Operations (OMOs): These include both, outright purchase and sale of government
securities, for injection and absorption of durable liquidity, respectively.

Market Stabilisation Scheme (MSS): This instrument for monetary management was introduced in
2004. Surplus liquidity of a more enduring nature arising from large capital inflows is absorbed
through sale of short-dated government securities and treasury bills. The cash so mobilised is held
in a separate government account with the Reserve Bank.

Q Source: Basics: Monetary policy

12 Natural rate of population increase per thousand was highest in India during which of the following
decades?

A. Post-liberalization period
B. Post-Bangladesh war period
C. Decade immediately after India’s independence
D. Post-Kargil war period

Correct Answer : B

Answer Justification :

Justification: During 1971-1981 natural increase rate per thousand was around 22. It was a period
of rapid growth of population.

Post-liberalization it hovered around 17-20.

Immediately after independence it was in the range of 13-19.

Post-Kargil war it was around 17. After 2001 population growth slowed down.

To sum it percentage terms, during 1921 to 1951 A.D., there was an increase in population by 12%
and from 1951 to 1981, the growth showed an increase by 24.75%.

Q Source: Based on UPSC past year papers

13 World Energy Outlook (WEO) and Energy Access outlook (EAO) is released by

8
Total Marks : 200
Online Prelims TEST - 34 (TEXTBOOK)
( InsightsIAS Mock Test Series for UPSC Preliminary Exam 2020 ) Mark Scored : 0

A. UN Environment Programme
B. IMF
C. World Bank
D. International Energy Agency (IEA)

Correct Answer : D

Answer Justification :

Justification: International Energy Agency (IEA) releases World Energy Outlook (WEO) and
Energy Access outlook.

The World Energy Outlook series is a leading source of strategic insight on the future of energy and
energy-related emissions, providing detailed scenarios that map out the consequences of different
energy policy and investment choices.

Deep disparities define today’s energy world: oil markets and geopolitical tensions, carbon
emissions and climate targets, the promise of energy for all and the lack of electricity access for 850
million people around the world.

World Energy Outlook 2019 explores these widening fractures in detail. It explains the impact of
today’s decisions on tomorrow’s energy systems, and describes a pathway that enables the world to
meet climate, energy access and air quality goals while maintaining a strong focus on the reliability
and affordability of energy for a growing global population.

Learning: IEA is an autonomous agency, founded in 1974 as per framework of the Organisation for
Economic Co-operation and Development (OECD). It seeks to promote energy security among its
member countries and ensure reliable, affordable and clean energy.

Its four focus area includes Energy Security, Economic Development, Environmental Awareness and
Engagement Worldwide. o India is an associated member to IEA.

Q Source: https://www.iea.org/reports/world-energy-outlook-2019

14 In Debt-for-nature swaps
1. Developed countries invest in climate mitigation measures as against giving financial support to
under-developed countries
2. A portion of a developing nation’s foreign debt is forgiven in exchange for local investments in
environmental conservation measures

Select the correct answer using the codes below.


A. 1 only
B. 2 only
C. Both 1 and 2
D. None of the above

9
Total Marks : 200
Online Prelims TEST - 34 (TEXTBOOK)
( InsightsIAS Mock Test Series for UPSC Preliminary Exam 2020 ) Mark Scored : 0

Correct Answer : B

Answer Justification :

Justification: Debt-for-nature swaps are financial transactions in which a portion of a developing


nation’s foreign debt is forgiven in exchange for local investments in environmental conservation
measures.

The debt-for-nature swaps concept was first given birth by Thomas Lovejoy of the World Wildlife
Fund in 1984 as an opportunity to deal with the problems of developing-nation indebtedness and its
consequent deleterious effect on the environment.

In the wake of the Latin American debt crisis that resulted in steep reductions to the environmental
conservation ability of highly indebted nations, Lovejoy suggested that ameliorating debt promoting
conservation could be done at the same time. Since the first swap occurred between Conservation
International and Bolivia in 1987, many national governments and conservation organizations have
engaged in debt-for-nature swaps. Most swaps occur in tropical countries, which contain many
diverse species of flora and fauna.

Learning: The financing mechanism for debt-for-nature swaps is an agreement among the
funder(s), the national government of the debtor country, and the conservation organization(s)
using the funds. The national government of the indebted country agrees to a payment schedule on
the amount of the debt forgiven, usually paid through the nation’s central bank, in local currency or
ponds.

Participation in debt-for-nature swaps has been restricted primarily to countries where the risk of
default on debt payments is high. In these circumstances, the funder can purchase the debt at well
below its face value.

In a commercial debt-for-nature swap or three-party debt-for-nature swap, a non-governmental


organization (NGO) acts as the funder/donor and purchases debt titles from commercial banks on
the secondary market.

Bilateral debt-for-nature swaps take place between two governments. In a bilateral swap, a creditor
country forgives a portion of the public bilateral debt of a debtor nation in exchange for
environmental commitments from that country.

Q Source:
https://www.sdfinance.undp.org/content/sdfinance/en/home/solutions/debt-for-nature-swaps.html

15 With reference to the Industrial Finance Corporation of India (IFCI), consider the following
statements.
1. IFCI is a Systemically Important Non-Deposit taking Non-Banking Finance Company (NBFC-ND-SI),
registered with the Reserve Bank of India.
2. The primary business of IFCI is to provide medium to long term financial assistance to the
manufacturing, services and infrastructure sectors.

Select the correct answer using the codes below.


A. 1 only
10
Total Marks : 200
Online Prelims TEST - 34 (TEXTBOOK)
( InsightsIAS Mock Test Series for UPSC Preliminary Exam 2020 ) Mark Scored : 0

B. 2 only
C. Both 1 and 2
D. None of the above

Correct Answer : C

Answer Justification :

Justification: IFCI Ltd. was set up in 1948 as Industrial Finance Corporation of India, a Statutory
Corporation, through `The Industrial Finance Corporation of India Act, 1948’ of Parliament to
provide medium and long term finance to industry. After repeal of this Act in 1993, IFCI became a
Public Limited Company registered under the Companies Act, 1956. The IFCI is now a Government
Company under the Companies Act, 2013.

IFCI is also a Systemically Important Non-Deposit taking Non-Banking Finance Company (NBFC-
ND-SI), registered with the Reserve Bank of India.

The primary business of IFCI is to provide medium to long term financial assistance to the
manufacturing, services and infrastructure sectors. Through its subsidiaries and associate
organizations, IFCI has diversified into a range of other businesses including broking, venture
capital, financial advisory, depository services, factoring etc.
As part of its development mandate, IFCI was one of the promoters of National Stock
Exchange (NSE), Stock Holding Corporation of India Ltd (SHCIL), Technical Consultancy
Organizations (TCOs) and social sector institutions like Rashtriya Gramin Vikas Nidhi (RGVN),
Management Development Institute (MDI) and Institute of Leadership Development (ILD).

Q Source: Based on past year papers UPSC

16 Consider the following statements.


1. Green India Mission (GIM) is one of the eight missions launched under the National Action Plan on
Climate Change (NAPCC).
2. GIM aims to increase forest-based livelihood incomes and enhance annual Carbon sequestration of
India.
3. GIM promotes adoption of improved fuelwood-use efficiency and alternative energy devices by
households.

Select the correct answer using the codes below.


A. 1 and 3 only
B. 1, 2 and 3
C. 2 and 3 only
D. 2 only

Correct Answer : B

Answer Justification :

11
Total Marks : 200
Online Prelims TEST - 34 (TEXTBOOK)
( InsightsIAS Mock Test Series for UPSC Preliminary Exam 2020 ) Mark Scored : 0

Justification: GIM is one of the eight missions launched under the National Action Plan on Climate
Change (NAPCC).

GIM, launched in February 2014, is aimed at protecting, restoring and enhancing India’s
diminishing forest cover and responding to climate change by a combination of adaptation and
mitigation measures.

Objectives of the Mission:

1. To protect, restore and enhance India's falling forest cover.

2. To respond to climate change through a combination of adaptation as well as mitigation


measures.

3. To increase forest-based livelihood incomes.

4. To enhance annual Carbon sequestration by 50 to 60 million tonnes in the year 2020.

It also engages in:

Management of public forest/ non-forests areas (taken up under the Mission) by the
community institutions
Adoption of improved fuelwood-use efficiency and alternative energy devices by project-area
households.
Diversification of forest-based livelihoods of about 3 million households living in and around
forests.

Q Source:
https://www.insightsonindia.com/wp-content/uploads/2020/03/INSTA-PT-2020-Exclusive-Environme
nt.pdf

17 Which of the following are State trees of one or more states of India?
1. Neem
2. Peepal
3. Deodar
4. Sandal

Select the correct answer using the codes below.


A. 1, 2 and 4 only
B. 4 only
C. 1 and 3 only
D. 1, 2, 3 and 4

Correct Answer : D

Answer Justification :

Learning: Neem: AP; Deodar: HP; Peepal: Bihar and Haryana; Sandal: Karnataka.

12
Total Marks : 200
Online Prelims TEST - 34 (TEXTBOOK)
( InsightsIAS Mock Test Series for UPSC Preliminary Exam 2020 ) Mark Scored : 0

This is a comprehensive list of State Animals, Birds, Trees and Flowers of India.

You should ideally research a bit on some important species mentioned here

http://www.frienvis.nic.in/KidsCentre/State-Animals-Birds-Trees-Flowers-of-India_1500.aspx

Questions on most of these species have been covered in the previous tests and will be covered in
upcoming tests.

Q Source: Species of importance

18 Consider the following statements.


1. All grants given to the State Governments/Union Territories are considered revenue expenditure
even though some of the grants may be used for creation of capital assets.
2. Revenue expenditure which results in the creation of capital assets is reduced from revenue deficit
to arrive at the Primary Deficit (PD).

Which of the above is/are correct?


A. 1 only
B. 2 only
C. Both 1 and 2
D. Neither 1 nor 2

Correct Answer : A

Answer Justification :

Learning: Broadly, the expenditure which does not result in creation of assets for the Government
of India, is treated as revenue expenditure.

All grants given to the State Governments/Union Territories and other parties are also treated as
revenue expenditure even though some of the grants may be used for creation of capital assets.

Revenue expenditure which results in the creation of capital assets is reduced from revenue deficit
to arrive at the effective revenue deficit (ERD).

Q Source: Basics: Indian economy

19 Following this report, the concept of a merit based modern Civil Service in India was introduced in
1854. The Report recommended that patronage based system of East India Company should be
replaced by a permanent Civil Service based on a merit based system with entry through competitive
examinations. The committee was

A. Raleigh Commission led by Curzon


B. Macaulay’s Report of the Select Committee of British Parliament
C. Ashlington Committee of the States
D. Hartog Committee led by Irwin

13
Total Marks : 200
Online Prelims TEST - 34 (TEXTBOOK)
( InsightsIAS Mock Test Series for UPSC Preliminary Exam 2020 ) Mark Scored : 0

Correct Answer : B

Answer Justification :

Justification: Civil Servants for the East India Company used to be nominated by the Directors of
the Company and thereafter trained at Haileybury College in London and then sent to India.
Following Lord Macaulay’s Report of the Select Committee of British Parliament, the concept of a
merit based modern Civil Service in India was introduced in 1854. The Report recommended that
patronage based system of East India Company should be replaced by a permanent Civil Service
based on a merit based system with entry through competitive examinations. For this purpose, a
Civil Service Commission was setup in 1854 in London and competitive examinations were started
in 1855. Initially, the examinations for Indian Civil Service were conducted only in London.
Maximum age was 23 years and minimum age was 18 years. The syllabus was designed such that
European Classics had a predominant share of marks. All this made it difficult for Indian
candidates. Nevertheless, in 1864, the first Indian, Shri Satyendranath Tagore brother of Shri
Rabindaranath Tagore succeeded.

Q Source:
https://www.upsc.gov.in/sites/default/files/History%20of%20the%20Commission%20final%20%281
%29_0.pdf

20 The East-West Industrial Corridor, that was recently in news, passes through which of these
protected areas?

A. Nokrek National Park


B. Namdapha National Park
C. Dachigam Wildlife Sanctuary
D. Pakke Tiger Reserve

Correct Answer : D

Answer Justification :

Justification: Arunachal Pradesh State government is planning to build a 692.7 km highway


through the Pakke Tiger Reserve (PTR) in East Kameng district. Named the East-West Industrial
Corridor, the highway aims to connect BhairabKunda in West Kameng district and Manmao in
Changlang district along Arunachal Pradesh’s border with Assam.

Pakke Tiger Reserve is also known as Pakhui Tiger Reserve. This Tiger Reserve has won India
Biodiversity Award 2016 in the category of 'Conservation of threatened species' for its Hornbill Nest
Adoption Programme.

It is bounded by Bhareli or Kameng River in the west and north, and by Pakke River in the east.

Neighbours: Papum Reserve Forest in Arunachal Pradesh, Assam’s Nameri National Park, Doimara
Reserve Forest and Eaglenest Wildlife Sanctuary. o The main perennial streams in the area are the

14
Total Marks : 200
Online Prelims TEST - 34 (TEXTBOOK)
( InsightsIAS Mock Test Series for UPSC Preliminary Exam 2020 ) Mark Scored : 0

Nameri, Khari and Upper Dikorai. West of Kameng River is Sessa Orchid Sanctuary.

Q Source: In news

21 Special Mention Accounts (SMA), seen in news recently, concern

A. High interest rate bearing accounts


B. Stressed assets
C. Accounts in violation of Basel norms
D. Accounts in banks with no risk of default

Correct Answer : B

Answer Justification :

Justification: The classification of Special Mention Accounts (SMA) was introduced by the RBI in
2014, to identify those accounts that has the potential to become an NPA/Stressed Asset.

Special Mention Accounts are those assets/accounts that shows symptoms of bad asset quality in
the first 90 days itself.

The Special Mention Accounts are usually categorized in terms of duration. ▪ SMA – NF: Non-
financial (NF) signals of stress ▪ SMA-0: Principal or interest payment not overdue for more than 30
days but account showing signs of incipient stress. ▪ SMA- 1: Principal or interest payment overdue
between 31-60days. ▪ SMA – 2: Principal or interest payment overdue between 61-90 days.

Q Source: In news

22 ANDREX project, recently in news, is concerned with understanding the link between

A. The Southern Ocean and the atmospheric carbon dioxide levels


B. The Troposphere and the exchange of gaseous material in the atmosphere
C. Water vapour content and rainfall in Western coasts of the Mid-latitudinal regions
D. Arctic weather and navigation of ships in the arctic region

Correct Answer : A

Answer Justification :

Justification: Scientists have discovered that, contrary to existing assumptions, biological


processes far out at sea are the most important factors determining how the ocean absorbs carbon
dioxide.

This enhances the understanding of the link between the Southern Ocean — next to Antarctica —
and the atmospheric carbon dioxide levels.

15
Total Marks : 200
Online Prelims TEST - 34 (TEXTBOOK)
( InsightsIAS Mock Test Series for UPSC Preliminary Exam 2020 ) Mark Scored : 0

Scientists studied data collected as part of the ANDREX project (Antarctic Deep water Rates of
Export) which measured the physical, biological, and chemical properties of the waters in the gyre
between 2008 and 2010.

About ANDREX project: The project seeks to assess the role of the Weddell gyre in driving the
southern closure of the meridional overturning circulation, in ventilating the deep global ocean, and
in sequestering carbon and nutrients in the global ocean abyss.

Significance: Carbon dioxide is absorbed in the surface oceans and stored in the deep seas,
gradually, over a timescale of 100s to 1,000s years. The Southern Ocean plays a critical role in how
the carbon dioxide is taken out of the atmosphere, and knowing how it functions helps scientists
understand this mechanism’s role during dramatic climate transitions in the past, such as the ice
ages, and better predict the current and future climate change.

Q Source: In news

23 The CDP India annual report has been released by CDP (Carbon Disclosure Project) India. The report
examines carbon reduction activities of companies. CDP is

A. a wing of the United Nations Environment Programme


B. a non-profit international organization
C. a mechanism under UNFCCC
D. a division of the Convention on Biological Diversity

Correct Answer : B

Answer Justification :

Justification: CDP is a not-for-profit that runs the global disclosure system for investors,
companies, cities, states and regions to manage their environmental impacts. Over the past 15
years we have created a system that has resulted in unparalleled engagement on environmental
issues worldwide.

It collects and collates self-reported environmental data in the world. It is aimed at measuring the
carbon reduction activities undertaken by different companies and firms operating in various
countries across the globe.

Based on CDP report, India is now among the top five countries globally when it comes to adopting
science based target initiatives (SBT) with as many as 38 Indian companies in 2019 committing to
going beyond policy requirements to plan urgent climate action, a significant rise from 25 firms in
2018.

Q Source: CDP report in news

24 Which of the following is/are the components of the Climate Change Performance Index (CCPI),
2019?
1. GHG Emissions

16
Total Marks : 200
Online Prelims TEST - 34 (TEXTBOOK)
( InsightsIAS Mock Test Series for UPSC Preliminary Exam 2020 ) Mark Scored : 0

2. Renewable Energy
3. Energy Use
4. Climate Policy

Select the correct answer using the codes below.


A. 1, 2 and 3 only
B. 1 and 3 only
C. 1, 2, 3 and 4
D. 2 and 4 only

Correct Answer : C

Answer Justification :

Justification: The latest edition of Climate Change Performance Index (CCPI) was recently
presented at the climate summit in Madrid.

Designed by the German environmental and development organisation Germanwatch e.V.


Published in cooperation with the NewClimate Institute and Climate Action Network
International and with financial support from Barthel Foundation.

Objective: To enhance transparency in international climate politics. First published in 2005 and an
updated version is presented at the UN Climate Change Conference annually.

In 2017, the methodology of the CCPI was revised and adapted to the new climate policy framework
of the Paris Agreement from 2015.

The CCPI was extended in order to include the measurement of a country’s progress towards the
Nationally Determined Contributions (NDCs) and the country’s 2030 targets.

The national performances are assessed based on 14 indicators in the following four categories:

1. GHG Emissions (weighting 40%).

2. Renewable Energy (weighting 20%).

3. Energy Use (weighting 20%).

4. Climate Policy (weighting 20%). • India, at rank 9, joins the top ten countries.

17
Total Marks : 200
Online Prelims TEST - 34 (TEXTBOOK)
( InsightsIAS Mock Test Series for UPSC Preliminary Exam 2020 ) Mark Scored : 0

Q Source: Important indices

25 Union micro, small and medium enterprises sector (MSME) ministry is planning to launch enterprise
development centres (EDCs) in every district. What role and functions have been envisaged for the
EDCs?
1. Offer enterprise development courses, vocational guidance and skill development for budding
entrepreneurs
2. Offer credit facilitation and export promotion services

Select the correct answer using the codes below.


A. 1 only
B. 2 only
C. Both 1 and 2
D. None of the above

Correct Answer : C

Answer Justification :

Justification: EDCs are aimed at developing a cadre of indigenous entrepreneurs in the MSMEs,
the EDCs will be similar to incubators for start-ups.

They shall be run by special purpose vehicles in partnership with the private sector, business
management organisations, local industry associations.

Key features, roles and functions of EDCs:

1. Offer “enterprise development courses, vocational guidance and skill development for budding
entrepreneurs”.

18
Total Marks : 200
Online Prelims TEST - 34 (TEXTBOOK)
( InsightsIAS Mock Test Series for UPSC Preliminary Exam 2020 ) Mark Scored : 0

2. Have “enterprise clinics” for struggling firms, which the government hopes will reduce the
number of small businesses falling into a debt trap financed by bank loans.

3. Offer credit facilitation and syndication, export promotion and supplier inclusion.

4. Incentives and loans by the government for the sector will also flow through the EDCs, after
determining the capabilities of the firm through set parameters.

Q Source:
https://www.insightsonindia.com/wp-content/uploads/2020/03/INSTA-PT-2020-Exclusive-Economy.p
df

26 Under the Wild Life (Protection) Act, 1972, Wildlife Crime Control Bureau (WCCB) is mandated to
1. Establish a centralized wildlife crime data bank
2. Lay down regulations on the trade of species listed under various international or multi-lateral
wildlife conservation agreements of India
3. Assist and advise the Customs authorities in inspection of the consignments of flora & fauna

Select the correct answer using the codes below.


A. 1 and 3 only
B. 1, 2 and 3
C. 1 only
D. 2 and 3 only

Correct Answer : A

Answer Justification :

Justification Wildlife Crime Control Bureau is a statutory multi-disciplinary body established by the
Government of India under the Ministry of Environment and Forests, to combat organized wildlife
crime in the country.

Under Section 38 (Z) of the Wild Life (Protection) Act, 1972, it is mandated:

to collect and collate intelligence related to organized wildlife crime activities and to disseminate
the same to State and other enforcement agencies for immediate action so as to apprehend the
criminals.

to establish a centralized wildlife crime data bank.


co-ordinate actions by various agencies in connection with the enforcement of the provisions
of the Act.
assist foreign authorities and international organization concerned to facilitate coordination
and universal action for wildlife crime control.
capacity building of the wildlife crime enforcement agencies for scientific and professional
investigation into wildlife crimes and assist State Governments to ensure success in
prosecutions related to wildlife crimes.
advise the Government of India on issues relating to wildlife crimes having national and

19
Total Marks : 200
Online Prelims TEST - 34 (TEXTBOOK)
( InsightsIAS Mock Test Series for UPSC Preliminary Exam 2020 ) Mark Scored : 0

international ramifications, relevant policy and laws.


Assist and advise the Customs authorities in inspection of the consignments of flora & fauna
as per the provisions of Wild Life Protection Act, CITES and EXIM Policy governing such
anitem.

Q Source: Insights current events: Environment Compilation

27 'Operation Twist', recently seen in news, concerned

A. Flattening the bond yield curve


B. Foreign investment regulation
C. Fiscal stimulus to selected sectors
D. Easing of interest rates

Correct Answer : D

Answer Justification :

Justification: RBI launched US-style 'Operation Twist' to bring down interest rates. ‘Operation
Twist’ is when the central bank uses the proceeds from sale of short-term securities to buy long-
term government debt papers, leading to easing of interest rates on the long-term papers.

The objective behind such an operation is management of the yield curve. It will help to make loans
less expensive with those looking to buy homes, cars and make savings less desirable as it doesn’t
pay much interest.

Other central banks, including the US Federal Reserve, have used similar measures. This is the first
time RBI has undertaken such an unconventional policy measure with the aim of flattening the yield
curve by lowering longer rates to boost lending and growth

Q Source: In news

28 With reference to Core Investment Companies (CIC), consider the following statements.
1. CICs are those non-banking financial companies of which a majority of shares are owned by
Infrastructure trusts Infrastructure funds.
2. As per RBI guidelines, CICs cannot engage in a business of acquisition of shares and securities.
3. CICs are allowed to accept public funds.

Select the correct answer using the codes below.


A. 1, 2 and 3
B. 1 and 2 only
C. 2 only
D. 3 only

Correct Answer : D

20
Total Marks : 200
Online Prelims TEST - 34 (TEXTBOOK)
( InsightsIAS Mock Test Series for UPSC Preliminary Exam 2020 ) Mark Scored : 0

Answer Justification :

Justification: CICs are non-banking financial companies with asset size of ₹100 crore and above
which carry on the business of acquisition of shares and securities, subject to certain conditions.

CICs, which are allowed to accept public funds, hold not less than 90% of their net assets in the
form of investment in equity shares, preference shares, bonds, debentures, debt or loans in group
companies.

Investments of CIC in the equity shares (including instruments compulsorily convertible into equity
shares within a period not exceeding 10 years from the date of issue) in group companies
constitutes not less than 60% of its net assets.

Exemption: CICs having asset size of below Rs 100 crore are exempted from registration and
regulation from the RBI, except if they wish to make overseas investments in the financial sector.

Learning: “Public funds" includes funds raised either directly or indirectly through public deposits,
inter-corporate deposits, bank finance and all funds received from outside sources such as funds
raised by issue of Commercial Papers, debentures etc. but excludes funds raised by issue of
instruments compulsorily convertible into equity shares within a period not exceeding 10 years from
the date of issue

Q Source: Insights: Economy compilation

29 The Basel Ban Amendment (1995), which became an international law in 2019, deals with
1. Global waste dumping prohibition
2. Movement of radioactive waste

Which of the above is/are correct?


A. 1 only
B. 2 only
C. Both 1 and 2
D. Neither 1 nor 2

Correct Answer : A

Answer Justification :

Justification: The 1995 Basel Ban Amendment, a global waste dumping prohibition, has become
an international law after Croatia (97th country to ratify) ratified it on September 6, 2019.

It will become a new Article in the Convention and will enter into force in the 97 countries after 90
days — on December 5.

It was adopted by the parties to the Basel Convention in 1995.


It aims to protect human health and the environment against the adverse effects of hazardous
wastes.

21
Total Marks : 200
Online Prelims TEST - 34 (TEXTBOOK)
( InsightsIAS Mock Test Series for UPSC Preliminary Exam 2020 ) Mark Scored : 0

The amendment prohibits all export of hazardous wastes, including electronic wastes and obsolete
ships from 29 wealthiest countries of the Organization of Economic Cooperation and Development
(OECD) to non-OECD countries. It does not address the movement of radioactive waste.

Learning: Basel Convention — Control of Transboundary Movements of Hazardous Wastes and


Their Disposal: o Opened for signature on 22 March 1989 entered into force on 5 May 1992 o
Parties — 187.

It is an international treaty that was designed to reduce the movements of hazardous waste
between nations, and specifically to prevent transfer of hazardous waste from developed to less
developed countries (LDCs).

Q Source: Insights current events: Environment Compilation

30 Global Coalition to protect Pollinators is an initiative of

A. United Nations Environment Programme


B. International Union for Conservation of Nature
C. Conference of the Parties – Convention of Biological Diversity
D. World Wildlife Fund

Correct Answer : C

Answer Justification :

Justification: The initiative to form a coalition was taken by the Netherlands on December 12,
2016 at the Conference of the Parties – Convention of Biological Diversity held in Mexico.

It was formed to follow up on the findings of IPBES Assessment on Pollinators, Pollination and Food
Production, which found that many of the world’s pollinator species are on the decline.

Joining the coalition means adopting the following measures:

Taking action to protect pollinators and their habitats by developing and implementing
national pollinator strategies.
Sharing experience and lessons learnt in developing and implementing national pollinator
strategies, especially knowledge on new approaches, innovations and best practices.
Reaching out to seek collaboration with a broad spectrum of stakeholders—countries as well
as businesses, NGOs, farmers and local communities.
Developing research on pollinator conservation. o Supporting and collaborating with each
other—and those parties that are willing to join the coalition.

Q Source: Insights current events: Environment Compilation

31 The National Anti-Profiteering Authority (NAA) has been constituted under

A. Central Goods and Services Tax Act, 2017


22
Total Marks : 200
Online Prelims TEST - 34 (TEXTBOOK)
( InsightsIAS Mock Test Series for UPSC Preliminary Exam 2020 ) Mark Scored : 0

B. One Hundred Eighteenth Constitutional Amendment Act, 2018


C. Companies Act, 2013
D. A resolution of the Cabinet, 2018

Correct Answer : A

Answer Justification :

Justification: The National Anti-Profiteering Authority (NAA) has been constituted under Section
171 of the Central Goods and Services Tax Act, 2017. It is to ensure the reduction in rate of tax or
the benefit of input tax credit is passed on to the recipient by way of commensurate reduction in
prices.

The Authority’s core function is to ensure that the benefits of the reduction is GST rates on goods
and services made by GST Council and proportional change in the Input tax credit passed on to the
ultimate consumers and recipient respectively by way of reduction in the prices by the suppliers.

The National Anti-profiteering Authority shall be headed by a senior officer of the level of a
Secretary to the Government of India and shall have four technical members from the Centre and/or
the States.

Powers and functions of the authority:

In the event the National Anti-profiteering Authority confirms the necessity of applying anti-
profiteering measures, it has the power to order the business concerned to reduce its prices or
return the undue benefit availed along with interest to the recipient of the goods or services. o If
the undue benefit cannot be passed on to the recipient, it can be ordered to be deposited in the
Consumer Welfare Fund. o In extreme cases the National Anti-profiteering Authority can impose a
penalty on the defaulting business entity and even order the cancellation of its registration under
GST.

Q Source:
https://www.insightsonindia.com/wp-content/uploads/2020/03/INSTA-PT-2020-Exclusive-Economy.p
df

32 National Scheduled Tribes Finance and Development Corporation (NSTFDC), a not-for-profit


company, provides financial assistance for
1. Skill and entrepreneurial development of the target group
2. Procurement and marketing minor forest produce so as to avoid the distress sale of the produce.

Select the correct answer using the codes below.


A. 1 only
B. 2 only
C. Both 1 and 2
D. Neither 1 nor 2

23
Total Marks : 200
Online Prelims TEST - 34 (TEXTBOOK)
( InsightsIAS Mock Test Series for UPSC Preliminary Exam 2020 ) Mark Scored : 0

Correct Answer : C

Answer Justification :

Justification: With a view to pay a focused attention and accelerate the pace of economic
development of scheduled tribes, the erstwhile National Scheduled Castes and Scheduled Tribes
Financial Development Corporation was bifurcated and National Scheduled Tribes Finance and
Development Corporation (NSTFDC) was set up in 2001 under the Ministry of Tribal Affairs.

The NSTFDC has been granted license under Section 25 of the Companies Act (A Company not for
profit). In order to achieve the mandate set for the NSTFDC, (for undertaking self-employment
ventures/activities) financial assistance is extended by NSTFDC to the scheduled tribes, who are
having annual family income upto double the poverty line.

NSTFDC also provides financial assistance as grant for skill and entrepreneurial development of the
target group. The financial assistance is channelized through government owned agencies
nominated by the respective ministries/state governments and union territory administrations.

NSTFDC also provides financial assistance for procurement and marketing


minor forest produce so as to avoid the distress sale of produce/ products by the
scheduled tribes.

The beneficiary (ies) should belong to ST community and annual family income of the beneficiary
(ies) should not exceed double the poverty line (DPL) income limit (presently DPL is ₹ 39,500/- per
annum for the rural areas and ₹ 54,500/- per annum for the urban areas). In the case of Self Help
Groups (SHGs), all the members of the SHG should belong to the ST community having annual
family income upto Double the Poverty Line (DPL). Similarly for cooperative society, loan is
extended to the eligible STs through cooperative society having minimum 80 per cent or more STs
as members.

Q Source: Ch on Social Welfare: IYB 2020

33 Consider the following about the Earth System Science Organization (ESSO).
1. It operates as an executive arm of the Ministry of Environment, Forest and Climate Change
(MoEFCC) for its policies and programmes.
2. It deals with atmospheric and climate science, including polar science and the cryo-sphere.
3. ESSO is responsible for the development of technology towards the sustainable exploration and
exploitation of marine resources.
4. It primarily aims to develop and improve capability to forecast, inter alia, climate and hazard
related phenomena for social, economic and environmental benefits.

Select the correct answer using the codes below.


A. 1, 2 and 4 only
B. 2, 3 and 4 only
C. 1 and 3 only
D. 1, 2, 3 and 4

24
Total Marks : 200
Online Prelims TEST - 34 (TEXTBOOK)
( InsightsIAS Mock Test Series for UPSC Preliminary Exam 2020 ) Mark Scored : 0

Correct Answer : B

Answer Justification :

Justification: The Earth System Science Organization (ESSO) operates as an executive arm
of the Ministry of Earth Science - MoES - for its policies and programmes.

It provides overall direction for the centres/units and reviews the implementation of the
programmes. The ESSO was established in 2007 as a virtual organization, bringing all
meteorological and ocean development activities under one umbrella, recognizing the importance of
strong coupling among various components of the earth viz., atmosphere, oceans, cryo-sphere and
geo-sphere.

It has four major branches of earth sciences, viz.: (i) ocean science and technology (ii) atmospheric
and climate science (iii) geo-science and technology and (iv) polar science and cryo-sphere. The sole
purpose of the endeavour was to address holistically various aspects relating to earth processes for
understanding the variability of earth system.

S3 and S4: The organisation is primarily aimed to develop and improve capability to forecast,
weather, climate and hazard related phenomena for social, economic and environmental
benefits including addressing aspects relating to climate change science, and climate
services. ESSO is also responsible for development of technology towards the exploration and
exploitation of marine resources in a sustainable way for the socio-economic benefit of the
society by taking into account the global developments in the field of marine environment.
One of the mandates of ESSO is also to promote research in polar science of both Antarctic
and Arctic regions to understand the various phenomenon and processes of these
regions on global climate and weather, in particularly on the Indian Ocean.

Q Source: MoES Website

34 With reference to Small Farmers’ Agri-Business Consortium (SFAC), consider the following
statements.
1. SFAC is an Autonomous Society promoted by Department of Agriculture, Cooperation and Farmers
Welfare, Ministry of Agriculture and Farmers Welfare.
2. It facilitates agri-business ventures by catalysing private investment through Venture Capital
Assistance (VCA) Scheme in association with financial institutions.
3. It is responsible for the implementation of National Agriculture Market (e-NAM) Electronic Trading
platform.

Select the correct answer using the codes below.


A. 1 only
B. 1 and 2 only
C. 1, 2 and 3
D. 2 only

Correct Answer : C

25
Total Marks : 200
Online Prelims TEST - 34 (TEXTBOOK)
( InsightsIAS Mock Test Series for UPSC Preliminary Exam 2020 ) Mark Scored : 0

Answer Justification :

Justification: The Government established Small Farmers’ Agri-Business Consortium (SFAC) as a


Society in January 1994 to facilitate agri-business ventures by catalysing private investment
through Venture Capital Assistance (VCA) Scheme in close association with financial institutions.
SFAC is an Autonomous Society promoted by Department of Agriculture, Cooperation and Farmers
Welfare, Ministry of Agriculture and Farmers Welfare, Govt. of India" It is registered under
Societies Registration Act 1860.

The role of State SFACs is to aggressively promote agribusiness project development in their
respective States.

Management: The Society is governed by Board of Management which is chaired, exofficio, by


Union Minister for Agriculture and Farmers Welfare as the President and the Secretary,
Department of Agriculture, Cooperation and Farmers Welfare, Government of India, is the ex-officio
Vice-President.

The main functions of SFAC are:

Promotion of development of small agribusiness through VCA scheme.


Helping formation and growth of Farmer Producer Organizations (FPOs) / Farmer Producer
Companies (FPCs).
Improving availability of working capital and development of business activities of FPOs/FPCs
through Equity Grant and Credit Guarantee Fund Scheme. o Implementation of National
Agriculture Market (e-NAM) Electronic Trading platform.

Q Source:
https://www.insightsonindia.com/wp-content/uploads/2020/03/INSTA-PT-2020-Exclusive-Economy.p
df

35 Which of the following phenomenon is observed due to Doppler Effect?


1. Fluctuations in the pitch of a siren based on its movement
2. Redshift or blueshift
3. Fission processes in Nuclear Reactors

Select the correct answer using the codes below.


A. 1 and 2 only
B. 3 only
C. 2 and 3 only
D. 1, 2 and 3 only

Correct Answer : A

Answer Justification :

Justification: The Doppler Effect (or Doppler shift) is the change in frequency of a wave (or other
periodic event) for an observer moving relative to its source.

26
Total Marks : 200
Online Prelims TEST - 34 (TEXTBOOK)
( InsightsIAS Mock Test Series for UPSC Preliminary Exam 2020 ) Mark Scored : 0

For e.g. it causes a car engine or siren to sound higher in pitch when it is approaching than when it
is receding.

Redshift and blueshift describe how light shifts toward shorter or longer wavelengths as objects in
space (such as stars or galaxies) move closer or farther away from us. The concept is key to
charting the universe's expansion.

The Doppler Effect for electromagnetic waves such as light is of great use in astronomy and results
in either a so-called redshift or blueshift. It has been used to measure the speed at which stars and
galaxies are approaching or receding from us; that is, their radial velocities.

S3 is irrelevant to Doppler effect.

Q Source: Based on UPSC papers

36 In the Seventeenth Lok Sabha, the largest number of state constituencies represented are from

A. Uttar Pradesh and Maharashtra


B. Uttar Pradesh and West Bengal
C. Maharashtra and Rajasthan
D. Uttar Pradesh and Rajasthan

Correct Answer : A

Answer Justification :

Learning: UP has 80 seats, MH has 48, WB has 42 and Rajasthan only 25. The entire list can be
accessed here http://164.100.47.194/Loksabha/Members/StatewiseList.aspx

The division of constituencies or the number of seats from a state is only dependent on its
population.

Of all the UTs, NCT Delhi has the largest 7 seats, all the other UTs have only one seat each.

Q Source: Basics: Polity

37 With reference to National Policy on Biofuels, 2018, consider the following statements.
1. The policy employs different financial incentives for different category of biofuels as identified in
the policy document.
2. The Policy allows use of surplus food grains for production of ethanol.
3. The Policy indicates a viability gap funding scheme for certain ethanol Bio refineries.

Select the correct answer using the codes below.


A. 2 only
B. 1 and 3 only
C. 3 only
D. 1, 2 and 3
27
Total Marks : 200
Online Prelims TEST - 34 (TEXTBOOK)
( InsightsIAS Mock Test Series for UPSC Preliminary Exam 2020 ) Mark Scored : 0

Correct Answer : D

Answer Justification :

Justification: The National Policy on Biofuels-2018 approved by the Government envisages an


indicative target of 20% blending of ethanol in petrol and 5% blending of bio-diesel in diesel by
2030. National Policy on biofuels- salient features:

Categorization: The Policy categorises biofuels as “Basic Biofuels” viz. First Generation (1G)
bioethanol & biodiesel and “Advanced Biofuels” – Second Generation (2G) ethanol, Municipal
Solid Waste (MSW) to drop- in fuels, Third Generation (3G) biofuels, bio-CNG etc. to enable
extension of appropriate financial and fiscal incentives under each category.
Scope of raw materials: The Policy expands the scope of raw material for ethanol production
by allowing use of Sugarcane Juice, Sugar containing materials like Sugar Beet, Sweet
Sorghum, Starch containing materials like Corn, Cassava, Damaged food grains like wheat,
broken rice, Rotten Potatoes, unfit for human consumption for ethanol production.
Protection to farmers: Farmers are at a risk of not getting appropriate price for their produce
during the surplus production phase. Taking this into account, the Policy allows use of surplus
food grains for production of ethanol for blending with petrol with the approval of National
Biofuel Coordination Committee.
Viability gap funding: With a thrust on Advanced Biofuels, the Policy indicates a viability gap
funding scheme for 2G ethanol Bio refineries of Rs.5000 crore in 6 years in addition to
additional tax incentives, higher purchase price as compared to 1G biofuels.
Boost to biodiesel production: The Policy encourages setting up of supply chain mechanisms
for biodiesel production from non-edible oilseeds, Used Cooking Oil, short gestation crops.

Q Source: In news frequently

38 Annual Survey of Industries is published by

A. Ministry of Heavy Industries and Public Enterprises


B. Ministry of Statistics and Programme Implementation
C. Ministry of Commerce
D. Department of Industrial Policy and Promotion

Correct Answer : B

Answer Justification :

Learning: The Annual Survey of Industries (ASI) is the principal source of industrial statistics in
India.

The Survey is conducted annually under the statutory provisions of the Collection of Statistics Act
2008.

The ASI extends to the entire country except the States of Arunachal Pradesh and Mizoram and

28
Total Marks : 200
Online Prelims TEST - 34 (TEXTBOOK)
( InsightsIAS Mock Test Series for UPSC Preliminary Exam 2020 ) Mark Scored : 0

Union territory of Lakshadweep.

It covers all factories registered under the sections 2(m) (i) and 2(m) (ii) of the Factories Act 1948
i.e. those factories employing 10 or more workers using power; and those employing 20 or more
workers without using power.

Q Source: Ch 1: Indian Economy: Ramesh Singh

39 Nilgiri Biosphere Reserve encompasses which of the following?


1. Bandipur National Park
2. Mudumalai National Park
3. Sathyamangalam wildlife sanctuary
4. Periyar National Park

Select the correct answer using the codes below.


A. 2, 3 and 4 only
B. 1 and 4 only
C. 1, 2 and 3 only
D. 2 and 4 only

Correct Answer : C

Answer Justification :

Justification: The Nilgiri Biosphere Reserve is an International Biosphere Reserve in the Western
Ghats and Nilgiri Hills ranges of South India. The Nilgiri Sub-Cluster is a part of the Western Ghats,
which was declared a World Heritage Site by UNESCO in 2012. It includes the Aralam, Mudumalai,
Mukurthi, Nagarhole, Bandipur and Silent Valley national parks, as well as the Wayanad and
Sathyamangalam wildlife sanctuaries.

A fascinating ecosystem of the hill ranges of Nilgiris and its surrounding environments covering a
tract of over 5000 square kilometers was constituted as Nilgiris Biosphere Reserve by UNESCO in
September 1986 under Man and Biosphere Programme. Nilgiris Biosphere Reserve is India's first
and foremost biosphere reserves with a heritage, rich in flora and fauna. Tribal groups like the
Badagas, Todas, Kotas, Irullas, Kurumbas, Paniyas, Adiyans, Edanadan Chettis, Allar, Malayan, etc.,
are native to the reserve.

Q Source: Protected areas of India

40 Partial Credit Guarantee Scheme was announced for

A. Below Poverty line households


B. Non-banking financial companies (NBFCs)
C. Software Parks
D. Small and Medium Enterprises

29
Total Marks : 200
Online Prelims TEST - 34 (TEXTBOOK)
( InsightsIAS Mock Test Series for UPSC Preliminary Exam 2020 ) Mark Scored : 0

Correct Answer : B

Answer Justification :

Justification: Partial Credit Guarantee Scheme for NBFCs got cabinet nod.

It allows for purchase of high-rated pooled assets from financially-sound nonbanking financial
companies (NBFCs) and housing finance companies (HFCs) by public sector banks (PSBs).

The scheme would cover NBFCs and HFCs that might have slipped into "Special Mention Accounts
(SMA)-0" category during the one-year period prior to August 1, 2018, and asset pools rated
"BBB+" or higher.

The window for one-time partial credit guarantee offered by the government would remain open till
June 30, 2020 or till such date by which Rs 1 lakh crore worth of assets get purchased by the banks,
whichever is earlier.

The Finance Minister would have the power to extend the validity of the scheme by up to three
months by taking into account its progress.

Q Source:
https://www.insightsonindia.com/wp-content/uploads/2020/03/INSTA-PT-2020-Exclusive-Economy.p
df

41 Fold Mountains are caused by large-scale earth's movements, when stresses caused may be due to
the
1. Increased load of the underlying rocks
2. Flow movements in the mantle
3. Magnetic intrusions into the crust
4. Expansion or contraction of some part of the crust

Select the correct answer using the codes below.


A. 1 and 4 only
B. 1 only
C. 2, 3 and 4 only
D. 1, 2, 3 and 4

Correct Answer : D

Answer Justification :

Justification and Learning: Fold Mountains are caused by large-scale earth's movements, when
stresses may be due to the increased load of the underlying rocks, flow movements in the mantle,
magnetic intrusions into the crust, or the expansion or contraction of some part of the earth.

When such stresses are initiated, the rocks are subjected to compressive forces that produce

30
Total Marks : 200
Online Prelims TEST - 34 (TEXTBOOK)
( InsightsIAS Mock Test Series for UPSC Preliminary Exam 2020 ) Mark Scored : 0

wrinkling or folding along the lines of weakness.

Fold mountains are created through a process called orogeny. An orogenic event takes millions of
years to create a fold mountain, but you can mimic it in seconds. Cover a table with a tablecloth, or
place a rug flat on the floor. Now push the edge of the tablecloth or rug—wrinkles will develop and
fold on top of each other.

The huge difference between the rock folds and cloth folds is that in the tabletop experiment, the
table itself does not fold. In the creation of fold mountains, Earth’s crust itself is warped into folded
forms.

Fold mountains are often associated with continental crust. They are created at convergent plate
boundaries, sometimes called continental collision zones or compression zones. Convergent plate
boundaries are sites of collisions, where tectonic plates crash into each other. Compression
describes a set of stresses directed at one point in a rock or rock formation.

Since the rock strata have been elevated to great heights, sometimes measurable in kilometres,
Fold Mountains may be called mountains of elevation. The Fold Mountains are also closely
associated with volcanic activity.

Q Source:https://www.nationalgeographic.org/encyclopedia/fold-mountain/

42 Ecotones can exist where two different __________ bodies meet.

A. Forest
B. Grassland
C. Water
D. All of the above

Correct Answer : D

Answer Justification :

Learning: Ecotone, a transitional area of vegetation between two different plant communities, such
as forest and grassland. It has some of the characteristics of each bordering biological community
and often contains species not found in the overlapping communities.

An ecotone may exist along a broad belt or in a small pocket, such as a forest clearing, where
two local communities blend together.
The influence of the two bordering communities on each other is known as the edge effect.
Ecotones also appear where one body of water meets another (e.g., estuaries and lagoons) or
at the boundary between the water and the land (e.g., marshes).

Q Source:Based on UPSC papers

43 UN Environment Programme (UNEP) member states recently adopted the “Colombo Declaration”,
which calls for tackling the global challenge of halving the waste of

31
Total Marks : 200
Online Prelims TEST - 34 (TEXTBOOK)
( InsightsIAS Mock Test Series for UPSC Preliminary Exam 2020 ) Mark Scored : 0

A. Sulphur
B. Nitrogen
C. Ozone
D. Lead

Correct Answer : B

Answer Justification :

Learning: At a time when the world grapples with the menace of air pollution killing 7 million
people prematurely every year, Sri Lanka, with support from the UN Environment Programme
(UNEP), convened a two-day event at which member states came together to adopt what is being
called the “Colombo Declaration” with an ambition to halve nitrogen waste by 2030.

While a critical element for building structures of living organisms and an essential element for the
survival of all living things, nitrogen overuse has negative impacts on the planet, biodiversity and is
a contributor to the climate crisis.

The Colombo Declaration calls upon UN agencies, other international organizations, development
partners, philanthropic agencies, academic and civil society organizations to support its
implementation. It further urges countries to conduct a comprehensive assessment on nitrogen
cycling covering policy, implementation, regulation, and scientific aspects at a national level plus
sensitize the citizens to understand the natural nitrogen cycle and how human impacts alter its
balance.

Q
Source:https://www.unenvironment.org/news-and-stories/press-release/colombo-declaration-calls-ta
ckling-global-nitrogen-challenge

44 The Preamble to the Indian Constitution is based on the

A. ‘Outcome Paper’ of the Lahore Session of the Indian National Congress, 1930
B. Prelude of the Government of India Act, 1935
C. ‘Objectives Resolution’ adopted by the Constituent Assembly in 1946
D. Preamble of the Nehru Report produced in 1928

Correct Answer : C

Answer Justification :

Learning: ‘Preamble’ refers to the introduction or preface to the Constitution. It contains the
summary or essence of the Constitution.

The Preamble to the Indian Constitution is based on the ‘Objectives Resolution’, drafted and moved
by Pandit Nehru, and adopted by the Constituent Assembly. It has been amended by the 42nd
Constitutional Amendment Act (1976), which added three new words—socialist, secular and

32
Total Marks : 200
Online Prelims TEST - 34 (TEXTBOOK)
( InsightsIAS Mock Test Series for UPSC Preliminary Exam 2020 ) Mark Scored : 0

integrity.

Q Source: Chapter 4: Indian Polity: M Laxmikanth

45 'Sweat labour' implies goods produced by

A. Labourers trafficked from other nations


B. Labourers working in inhuman or unhealthy working conditions
C. Labour working in the unorganized sector which is not regulated by the state
D. Children of more than 14 years of age but less than 18 years

Correct Answer : B

Answer Justification :

Learning:Sweatshop (or sweat factory) is a term for a workplace with very poor, socially
unacceptable or illegal working conditions. The work may be difficult, dangerous, climatically
challenging or underpaid. Workers in sweatshops may work long hours with low pay, regardless of
laws mandating overtime pay or a minimum wage; child labor laws may also be violated.

The Fair Labor Association's "2006 Annual Public Report" inspected factories for FLA compliance in
18 countries including Bangladesh, El Salvador, Colombia, Guatemala, Malaysia, Thailand, Tunisia,
Turkey, China, India, Vietnam, Honduras, Indonesia, Brazil, Mexico, and the US.

The U.S. Department of Labor's "2015 Findings on the Worst Forms of Child Labor" found that "18
countries did not meet the International Labour Organization's recommendation for an adequate
number of inspectors."

Q Source:Based on UPSC papers

46 Which of the following processes help in the concentration of minerals on earth?


1. Hydrothermal deposits
2. Weathering
3. Water erosion

Select the correct answer using the codes below.


A. 1 and 2 only
B. 2 only
C. 2 and 3 only
D. 1, 2 and 3

Correct Answer : D

Answer Justification :

33
Total Marks : 200
Online Prelims TEST - 34 (TEXTBOOK)
( InsightsIAS Mock Test Series for UPSC Preliminary Exam 2020 ) Mark Scored : 0

Justification: Statement 1: Hydrothermal deposits are produced when groundwater circulates to


depth and heats up either by coming near a hot igneous body at depth or by circulating to great
depth along the geothermal gradient.

Such hot water can dissolve valuable substances throughout a large volume of rock. As the hot
water moves into cooler areas of the crust, the dissolved substances are precipitated from the hot
water solution.

Statement 2: During chemical weathering and original body of rock is greatly reduced in volume by
the process of leaching, which removes ions from the original rock. Elements that are not leached
form the rock thus occur in higher concentration in the residual rock.

Statement 3: The velocity of flowing water determines whether minerals are carried in suspension
or deposited. For e.g. when the velocity of the water slows, large minerals or minerals with a higher
density are deposited.

Q Source: Based on UPSC papers

47 Carbon monoxide (CO) is one of the most serious air pollutants. It is released in air by
1. Automobile exhaust
2. Volcanic activity
3. Forest and bushfires

Select the correct answer using the codes below.


A. 1 and 2 only
B. 2 and 3 only
C. 1 and 3 only
D. 1, 2 and 3

Correct Answer : D

Answer Justification :

Learning:It is produced as a result of incomplete combustion of carbon. Carbon monoxide is mainly


released into the air by automobile exhaust. Other sources, which produce CO, involve incomplete
combustion of coal, firewood, petrol, etc.

Carbon monoxide is present in small amounts in the atmosphere, chiefly as a product of volcanic
activity but also from natural and man-made fires (such as forest and bushfires, burning of crop
residues, and sugarcane fire-cleaning).

Q Source:11th NCERT: Chemistry

48 The difference between Gross Domestic Product (GDP) and Net Domestic Product (NDP) will reduce
if

A. Exports grow faster than imports.


34
Total Marks : 200
Online Prelims TEST - 34 (TEXTBOOK)
( InsightsIAS Mock Test Series for UPSC Preliminary Exam 2020 ) Mark Scored : 0

B. The economy becomes more technologically advanced.


C. Natural resource gets depleted.
D. Bank loans become more expensive.

Correct Answer : B

Answer Justification :

Justification: NDP = GDP – Depreciation

So it is basically the GDP minus the total value of the ‘wear and tear’ (depreciation) that happened
in the assets while the goods and services were being produced.

The governments of the economies decide and announce the rates by which assets depreciate (done
in India by the Ministry of Commerce and Industry) and a list is published, which is used by the
different sections of the economy to determine the real levels of depreciations in different assets.

So, if the depreciation levels are low, it may show the achievements of the economy in the area of
research and development which have tried cutting the levels of depreciation in a historical time
period.

Q Source: Chapter 1: Indian Economy: Ramesh Singh

49 Which of the following is/are associated with the Chakiarkoothu form of dance?
1. Sangam texts
2. Mizhavu
3. Koothambalam

Select the correct answer using the codes below.


A. 1, 2 and 3
B. 1 and 3 only
C. 2 and 3 only
D. 1 and 2 only

Correct Answer : C

Answer Justification :

Learning: It is performed in the Koothambalam; a place inside Hindu temples specifically designed
for performing Kutiyattam and ChakyarKoothu. Ideally, the performance takes place in conjunction
with festivals, presented by members of the Chakyar community along with the
AmbalavasiNambiars.

It is a solo performance, by a narrator in a distinctive headgear and black moustache with his torso
smeared with sandalwood paste and red dots all over the body. The headgear resembles snake’s
hood, to symbolise the narration by Anantha, the thousand headed serpent.

35
Total Marks : 200
Online Prelims TEST - 34 (TEXTBOOK)
( InsightsIAS Mock Test Series for UPSC Preliminary Exam 2020 ) Mark Scored : 0

Q Source: Based on UPSC papers

50 Consider the following statements.


In Gautam Buddha’s hagiographies, symbols such as
1. the empty seat represents the mahaparinibbana of the Buddha.
2. the wheel represents the first sermon of the Buddha

Which of the above is/are correct?


A. 1 only
B. 2 only
C. Both 1 and 2
D. None of the above

Correct Answer : B

Answer Justification :

Justification: According to hagiographies, the Buddha attained enlightenment while meditating


under a tree.

The empty seat was meant to indicate the meditation of the Buddha, and the stupa was meant to
represent the mahaparinibbana.

The wheel stood for the first sermon of the Buddha, delivered at Sarnath because it was here that
he set the wheel of Dhamma in motion.

Q Source: 11thTamilNadu History Textbook

51 Among the following, which of the following Indian cities lies the most westward?

A. Gandhinagar
B. Ahmedabad
C. Bhuj
D. Rajkot

Correct Answer : C

Answer Justification :

Justification:

36
Total Marks : 200
Online Prelims TEST - 34 (TEXTBOOK)
( InsightsIAS Mock Test Series for UPSC Preliminary Exam 2020 ) Mark Scored : 0

Q Source:Map based questions: India

52 The August Movement of 1942 in India was a movement of

A. Peasants against Zamindars in all rural hinterlands


B. Bhils against dikus
C. The Indian National Army (INA) against invaders
D. The Indian National Congress (INC) after the failure of the Cripps mission

Correct Answer : D

Answer Justification :

Justification: It was the Quit India Movement.

In 1939 Indian nationalists were angry that British Governor-General of India, Lord Linlithgow, had
without consultation with them brought India into the war. The Muslim League supported the war,
but Congress was divided.

Thus, the All-India Congress Committee after the failure of the Cripps Mission launched a mass
protest demanding what Gandhi called "An Orderly British Withdrawal" from India.

Q Source: Based on UPSC papers

53 Which of the following types of forests is/are likely to contain hardwood trees?
1. Tropical Evergreen Forests
2. Tropical Deciduous Forests
3. Temperate Evergreen Forests

Select the correct answer using the codes below.

37
Total Marks : 200
Online Prelims TEST - 34 (TEXTBOOK)
( InsightsIAS Mock Test Series for UPSC Preliminary Exam 2020 ) Mark Scored : 0

A. 1 and 2 only
B. 2 only
C. 1 and 3 only
D. 1, 2 and 3

Correct Answer : D

Answer Justification :

Justification: Hardwood trees are useful for making furniture, transport and constructional
materials are likely to be found in all these forests.

Statement 1: The canopy tree species are mostly tall hardwoods with broad leaves like rosewood,
ebony, mahogany.

Statement 2: The hardwood trees found in these forests are sal, teak, neem and shisham.

Statement 3: They comprise both hard and soft wood trees like oak, pine, eucalyptus, etc.

Q Source: 7th Geography NCERT

54 Why did the historian Barani refuseto consider the state in India under Delhi Sultans as truly Islamic?
1. Regulations made by the Sultan supplemented the Muslim law.
2. State policy was not dictated or decided by the Muslims theologians.

Which of the above is/are correct?


A. 1 only
B. 2 only
C. Both 1 and 2
D. None

Correct Answer : C

Answer Justification :

Justification: The state under the Delhi sultan was Islamic only in a formal sense.

This means that the sultans did not allow any open violation of the Islamic law and appointed
Muslim divines to profitable offices of state and granted them revenue-free lands.

But the sultans did not allow the Muslim divines to dictate the policy of the state. though they could
not rule in complete disregard of the views and wishes of these theologians.

And also the sultans had to supplement the Muslim law by framing their own regulations. That is
why, the contemporary historian. Barani, refused to consider the state in India as truly Islamic, but
one based on worldly or secular considerations (jahandan).

38
Total Marks : 200
Online Prelims TEST - 34 (TEXTBOOK)
( InsightsIAS Mock Test Series for UPSC Preliminary Exam 2020 ) Mark Scored : 0

Q Source: Based on UPSC papers

55 The Constitution of India establishes a federal system of government. It contains most of the usual
features of a federation, but also of a unitary system. Which of the following signify the latter?
1. Division of powers between Central and State governments
2. Flexibility of the Constitution
3. Independent Judiciary
4. All-India Services

Select the correct answer using the codes below.


A. 1 and 2 only
B. 2 and 4 only
C. 1 and 3 only
D. 1, 2 and 3 only

Correct Answer : B

Answer Justification :

Justification: The usual features of a federation, viz., two government, division of powers, written
Constitution, supremacy of Constitution, rigidity of Constitution, independent judiciary and
bicameralism.

However, the Indian Constitution also contains a large number of unitary or non-federal features,
viz., a strong Centre, single Constitution, single citizenship, flexibility of Constitution, integrated
judiciary, appointment of state governor by the Centre, all-India services, emergency provisions,
and so on.

Q Source: Chapter 3: Indian Polity: M Laxmikanth

56 NASA’s proposed ‘PUNCH mission’ will study


1. Deviations in the orbit of asteroids orbiting earth
2. Change in the distribution of mass of the solar system affecting heavenly trajectories

Select the correct answer using the codes below.


A. 1 only
B. 2 only
C. Both 1 and 2
D. None of the above

Correct Answer : D

Answer Justification :

Justification: NASA has selected a US based Indian researcher to lead its PUNCH mission which

39
Total Marks : 200
Online Prelims TEST - 34 (TEXTBOOK)
( InsightsIAS Mock Test Series for UPSC Preliminary Exam 2020 ) Mark Scored : 0

will image the Sun.

About PUNCH (Polarimeter to Unify the Corona and Heliosphere):

It is focused on understanding the transition of particles from the Sun’s outer corona to the solar
wind that fills interplanetary space.

It will consist of a constellation of four microsatellites that through continuous 3D deep-field


imaging, will observe the corona and heliosphere as elements of a single, connected system.

The mission is expected to be launched in 2022.

Q Source:In news

57 The speed of sound in air is affected by


1. Temperature
2. Humidity
3. Density

Select the correct answer using the codes below.


A. 1 only
B. 2 and 3 only
C. 1, 2 and 3
D. 1 and 3 only

Correct Answer : C

Answer Justification :

Justification: The speed of sound is affected by temperature and humidity. For e.g. sound passes
through hot air faster than it passes through cold air because it is less dense.

Also, as sound propagates through air, the air absorbs energy from the sound wave, attenuating
(weakening) it.

The attenuation of sound in air is affected by the relative humidity. Dry air absorbs far more
acoustical energy than does moist air. This is because moist air is less dense than dry air (water
vapor weighs less than air).

Q Source: 10th Science NCERT

58 Special Drawing Rights (SDR) can be used to


1. Settle Balance of Payment transactions and augment Foreign Exchange Reserves
2. Bridge fiscal deficit and fund infrastructure projects

Select the correct answer using the codes below.


A. 1 only

40
Total Marks : 200
Online Prelims TEST - 34 (TEXTBOOK)
( InsightsIAS Mock Test Series for UPSC Preliminary Exam 2020 ) Mark Scored : 0

B. 2 only
C. Both 1 and 2
D. None of the above

Correct Answer : A

Answer Justification :

Justification: It cannot be used to fund infrastructure projects as it is not a currency. Same goes
for settling domestic financial bills of the government.

In some international treaties and agreements, SDRs are used to value penalties, charges or
prices.The Montreal Convention and other treaties also use SDRs in this way.

It also forms a part of India’s Forex reserves. However, the contribution is relatively small when
compared to currency reserves of foreign nations.

Q Source: 12th NCERT Macroeconomics

59 Black Foot disease is caused due to the use of water contaminated with

A. Arsenic
B. Cadmium
C. Lead
D. Mercury

Correct Answer : A

Answer Justification :

Learning: Drinking water rich in arsenic over a long period leads to arsenic poisoning or
arsenicosis, also known as the Black foot disease.

Drinking arsenic-rich water over a long period results in various health effects including skin
problems (such as colour changes on the skin, and hard patches on the palms and soles of the feet),
skin cancer, cancers of the bladder, kidney and lung, and diseases of the blood vessels of the legs
and feet, and possibly also diabetes, high blood pressure and reproductive disorders.

Absorption of arsenic through the skin is minimal and thus hand-washing, bathing, laundry, etc.
with water containing arsenic do not pose human health risks.

Q Source:Based on UPSC paper

60 Arrange the following ecosystems in decreasing order of net primary productivity:

A. Tropical seasonal forest – Tropical rain forest – Temperate Grassland – Woodland and

41
Total Marks : 200
Online Prelims TEST - 34 (TEXTBOOK)
( InsightsIAS Mock Test Series for UPSC Preliminary Exam 2020 ) Mark Scored : 0

Shrubland.
B. Tropical seasonal forest – Tropical rain forest – Woodland and Shrubland – Temperate
Grassland
C. Tropical rain forest – Temperate Grassland – Woodland and Shrubland – Tropical seasonal
forest.
D. Tropical rain forest – Tropical seasonal forest – Woodland and Shrubland – Temperate
Grassland.

Correct Answer : D

Answer Justification :

Justification:Net primary productivity, or NPP, is gross primary productivity minus the rate of
energy loss to metabolism and maintenance. In other words, it's the rate at which energy is stored
as biomass by plants or other primary producers and made available to the consumers in the
ecosystem.

Net primary productivity varies among ecosystems and depends on many factors. These include
solar energy input, temperature and moisture levels, carbon dioxide levels, nutrient availability, and
community interactions (e.g., grazing by herbivores)^22squared. These factors affect how many
photosynthesizers are present to capture light energy and how efficiently they can perform their
role.

In terrestrial ecosystems, primary productivity ranges from about 2000 g/m sq./year in highly
productive tropical forests and salt marshes to less than 100 g/m sq./year in some deserts.

The order can be logically arrived at by elimination.

Tropical rain forests receive overall more rainfall than seasonal forests and grasslands. So, (a) and
(b) are eliminated.

Also, a forest will certainly be more productive than grassland. So, (c) is also eliminated. Correct
answer will be (d).

Q Source:Basics: Ecology

61 Earthquakes can occur at


1. Divergent plate boundaries.
2. Ocean-ocean convergent plate boundaries.
3. Ocean-continent plate boundaries.
4. Transform boundaries.

Select the correct answer using the codes below.


A. 1, 3 and 4 only
B. 2 and 4 only
C. 1 only
D. 1, 2, 3 and 4

42
Total Marks : 200
Online Prelims TEST - 34 (TEXTBOOK)
( InsightsIAS Mock Test Series for UPSC Preliminary Exam 2020 ) Mark Scored : 0

Correct Answer : D

Answer Justification :

Justification: Most earthquakes occur at the boundaries where the plates meet. In fact, the
locations of earthquakes and the kinds of ruptures they produce help scientists define the plate
boundaries.

There are three types of plate boundaries: spreading zones, transform faults, and subduction
zones.
At spreading zones, molten rock rises, pushing two plates apart and adding new material at
their edges.

Divergent boundaries are those at which crustal plates move away from each other, such as at mid-
oceanic ridge.

The formation of new ocean crust that is pushed away from both sides of the ridge fault creates a
tensional setting that results in the formation of the graben.

Earthquakes are located along the normal faults that form the sides of the rift or beneath the floor
of the rift.

Transform faults are found where plates slide past one another. Shallow‐focus earthquakes occur
along transform boundaries where two plates move past each other.

Q Source: Chapter 3: Goh Cheng Leong - Certificate Physical and Human Geography

62 What was agreed upon in the Gandhi-Irwin Pact 1931?


1. Setting up of a responsible government for an independent dominion of India
2. Discontinuation of the civil disobedience movement by the Indian NationalCongress
3. Removal of the tax on salt
4. Participation by the Indian National Congress in the SecondRound Table Conference

Select the correct answer using the codes below.


A. 3 and 4 only
B. 2, 3 and 4 only
C. 1 and 2 only
D. 1, 2, 3 and 4

Correct Answer : B

Answer Justification :

Justification & Learning: Salient features of this pact were as following:

The Congress would participate in the Round Table Conference.


The Congress would discontinue the Civil Disobedience Movement.
43
Total Marks : 200
Online Prelims TEST - 34 (TEXTBOOK)
( InsightsIAS Mock Test Series for UPSC Preliminary Exam 2020 ) Mark Scored : 0

The Government would withdraw all ordinances issued to curb the Congress.
The Government would withdraw all prosecutions relating to offenses other than violent one.
The Government would release all persons undergoing sentences of imprisonment for their
activities in the civil disobedience movement.
Manufacture of salt by Indians will be allowed.

Q Source: Chapter 29: India’s Struggle for Independence: Bipin Chandra

63 The ‘Madrid Protocol’ that was adopted to address the deterrence of some countries from acceding to
the Madrid Agreement, 1891, relates to

A. Tackling aircraft hijacks


B. International Trademark System
C. Controlling transnational crimes
D. Movement of hazardous wastes

Correct Answer : B

Answer Justification :

Learning: The Madrid Protocol (Protocol), an international treaty, was adopted in 1989 in order to
remove the difficulties that were deterring some countries from acceding to the Madrid Agreement
(Agreement), the 1891 treaty that established the system for the international registration of
trademarks.

The Protocol, which has been in force since April 1, 1996, has become a convenient and economical
means of securing trademark registration in member countries in Asia, Africa, Europe, the Middle
East, the Pacific Rim and the Western Hemisphere.

The Trade Mark (Amendment) Act, 2010 enabled India to accede to the Madrid Protocol which is a
simple, facilitative and cost-effective system for registration of international trade marks.

You can understand more here http://www.wipo.int/madrid/en/how_madrid_works.html

Q Source: Chapter 19: IYB 2020

64 The Election Commission registers political parties for the purpose of elections and grants them
recognition as national or state parties on the basis of their

A. Membership size
B. Regional Presence
C. Poll performance
D. Party exchequer

Correct Answer : C

44
Total Marks : 200
Online Prelims TEST - 34 (TEXTBOOK)
( InsightsIAS Mock Test Series for UPSC Preliminary Exam 2020 ) Mark Scored : 0

Answer Justification :

Learning: The other parties that are not recognized by the ECI are simply declared as registered-
unrecognised parties.

The recognition granted by the Commission to the parties determines their right to certain
privileges like allocation of the party symbols, provision of time for political broadcasts on the state-
owned television and radio stations and access to electoral rolls.

Q Source: Chapter 65: Indian Polity: M Laxmikanth

65 An energy pyramid shows


1. Rates of energy flow through trophic levels
2. Absolute amounts of energy stored in each trophic level

Select the correct answer using the codes below.


A. 1 only
B. 2 only
C. Both 1 and 2
D. None of the above

Correct Answer : A

Answer Justification :

Justification:Energy pyramids represent energy flow through trophic levels. For instance, the
pyramid below shows gross productivity for each trophic level in the Silver Springs ecosystem. An
energy pyramid usually shows rates of energy flow through trophic levels, not absolute amounts of
energy stored. It can have energy units, such as kcal/m sq./yr or in terms of biomass as g/m sq./yr

Energy pyramids are always upright (biomass pyramids can be inverted), that is, narrower at each
successive level (unless organisms enter the ecosystem from elsewhere). This pattern reflects the
laws of thermodynamics, which tell us that new energy can't be created, and that some must be
converted to a not-useful form (heat) in each transfer.

45
Total Marks : 200
Online Prelims TEST - 34 (TEXTBOOK)
( InsightsIAS Mock Test Series for UPSC Preliminary Exam 2020 ) Mark Scored : 0

66 A receipt is considered a capital receipt if it satisfies which of these conditions?


1. The receiptcreates a liability for the government.
2. The receiptcauses a decrease in government assets.

Which of the above is/are correct?


A. 1 only
B. 2 only
C. Either 1 or 2
D. None of the above

Correct Answer : C

Answer Justification :

Justification: A receipt is a capital receipt if it satisfies any one of the two conditions:

(i) The receipts must create a liability for the government. For example, Borrowings are capital
receipts as they lead to an increase in the liability of the government. However, tax received is not a
capital receipt as it does not result in creation of any liability.

(ii) The receipts must cause a decrease in the assets. For example, receipts from sale of shares of
public enterprise is a capital receipt as it leads to reduction in assets of the government.

Since, both borrowing and sale of government bonds create liability, they are called capital receipts.

Q Source: Chapter on Public Finance: 12th Macroeconomics NCERT

67 The Comptroller and Auditor General of India can resign any time from his office by addressing the
resignation letter to the president. But, he can also be removed from the office in the same manner as
that of

A. A member of the UPSC


B. A Supreme Court Judge
C. Cabinet Secretary
D. Chairman, Public Accounts Committee

Correct Answer : B

Answer Justification :

Learning: The CAG is appointed by the president of India by a warrant under his hand and seal.

He can be removed by the president on the basis of a resolution passed to that effect by both the
Houses of Parliament with special majority, either on the ground of proved misbehaviour or
incapacity.

46
Total Marks : 200
Online Prelims TEST - 34 (TEXTBOOK)
( InsightsIAS Mock Test Series for UPSC Preliminary Exam 2020 ) Mark Scored : 0

Member of UPSC can be removed by the President based on a SC inquiry. It does not require
Parliament’s approval.

Q Source: Chapter 45: Indian Polity: M Laxmikanth

68 Diamond mines in India can be found in


1. Odisha
2. Madhya Pradesh
3. Rajasthan
4. Jharkhand

Select the correct answer using the codes below.


A. 1, 2 and 3 only
B. 3 and 4 only
C. 1 and 2 only
D. 1, 2, 3 and 4

Correct Answer : C

Answer Justification :

Learning: Diamond fields of India are grouped into four regions:

South Indian tract of Andhra Pradesh, comprising parts of Anantapur, Cuddapah, Guntur,
Krishna, Mahaboobnagar and Kurnool districts;
Central Indian tract of Madhya Pradesh, comprising Panna belt;
Behradin-Kodawali area in Raipur district and Tokapal, Dugapal, etc. areas in Bastar district
of Chhattisgarh; and
Eastern Indian tract mostly of Odisha, lying between Mahanadi and Godavari valleys.

Q Source: Chapter 19: IYB 2020

69 Which of the following may be categorized as primary producers?


1. Diatom
2. Bacteria
3. Lichens

Select the correct answer using the codes below.


A. 1 and 2 only
B. 1 and 3 only
C. 2 and 3 only
D. 1, 2 and 3

Correct Answer : D

47
Total Marks : 200
Online Prelims TEST - 34 (TEXTBOOK)
( InsightsIAS Mock Test Series for UPSC Preliminary Exam 2020 ) Mark Scored : 0

Answer Justification :

Learning: Primary producers, also called autotrophs, are organisms that can produce their own
food.There are many different types of primary producers out in the Earth's ecosystem at different
states.

S3: Fungi and other organisms that gain their biomass from oxidizing organic materials are called
decomposers and are not primary producers. However, lichens located in tundra climates are an
exceptional example of a primary producer that, by mutualistic symbiosis, combine photosynthesis
by algae (or additionally nitrogen fixation by cyanobacteria) with the protection of a decomposer
fungus.

S2: Also, plant-like primary producers (trees, algae) use the sun as a form of energy and put it into
the air for other organisms.There are of course H2O primary producers, including a form of
bacteria, and phytoplankton. As there are many examples of primary producers, two dominant types
are coral and one of the many types of brown algae, kelp.

In almost all cases these are photosynthetically active organisms.

S1: Diatomsare a major group of algae,specifically microalgae, found in the oceans, waterways and
soils of the world. Living diatoms make up a significant portion of the Earth's biomass: they
generate about 20 to 50 percent of the oxygen produced on the planet each year,take in over 6.7
billion metric tons of silicon each year from the waters in which they live,and contribute nearly half
of the organic material found in the oceans.

The shells of dead diatoms can reach as much as a half-mile (800m) deep on the ocean floor, and the
entire Amazon basin is fertilized annually by 27 million tons of diatom shell dust transported by
transatlantic winds from the African Sahara.

Q Source: 9th Standard ICSE Textbook: Unit I

70 The Rapid Action Force (RAF) of India does NOT generally deal with

A. UN Peace Keeping operations


B. Insurgency in North-east
C. Communal problems and militancy in certain states
D. Surgical operations like tackling the hijacking of aircraft

Correct Answer : D

Answer Justification :

Learning: It is National Security Guard (NSG) that deals with such operations as in option (d).

RAF was raised to deal with communal problems and continues to deal with militancy in J and K and
Punjab, insurgency in north-east, overseas deployment for UN Peace Keeping in Haiti, Bosnia and
Kosovo, rescue and relief (Odisha super cyclone, Gujarat and J&K earthquake).

48
Total Marks : 200
Online Prelims TEST - 34 (TEXTBOOK)
( InsightsIAS Mock Test Series for UPSC Preliminary Exam 2020 ) Mark Scored : 0

RAF dealt with mammoth human congregations (Amarnath Yatra, Jagannath Yatra, etc.) combating
naxalism in LWE affected states guarding of vital shrines in Ayodhya, Kashi, Mathura, Vaishno Devi
and Raghunath Temple etc.

It also dealt with Jammu law and order situation on the issue of revocation of transfer of land to Shri
Amarnath Shrine Board, dealt with serious law and order problem due to Gujjar agitation in
Rajasthan, dealt effectively with serious stone pelting menace in J and K, and monitored successful
completion of elections in various states.

Q Source: Chapter 19: IYB 2020

71 Patches of vegetation reduce noise levels considerably by engaging in sound


1. Absorption
2. Refraction
3. Deflection

Select the correct answer using the codes below.


A. 1 and 2 only
B. 2 and 3 only
C. 1 and 3 only
D. 1, 2 and 3

Correct Answer : D

Answer Justification :

Justification: Statement 1: Plant parts such as stems, leaves, branches, wood, etc. absorb sound.
The greater number of plants, the size of the plant and the surface area of the plant will all affect its
ability to absorb sound.

Statement 2: Sound waves can also be refracted. A good example of this is carpeting in a home that
reduces the echoes.

Plantings that cover surface areas help accomplish the same feat. For example, vines on walls and
the sides of buildings will help refract sound. Lawns, ground cover plantings and green walls are
excellent at refracting sound.

Statement 3: When sound hits something solid, it does not vibrate (because it is rigid). Sound waves
are reflected off the it and back toward the source.

Q Source:Based on UPSC papers

72 Consider the following about Akash missile system.


1. It has been indigenously developed.
2. It is a nuclear capable supersonic missile.
3. It can be launched from deep ocean waters and even from the edge of troposphere.

49
Total Marks : 200
Online Prelims TEST - 34 (TEXTBOOK)
( InsightsIAS Mock Test Series for UPSC Preliminary Exam 2020 ) Mark Scored : 0

Select the correct answer using the codes below.


A. 1 and 2 only
B. 2 and 3 only
C. 1 and 3 only
D. 1, 2 and 3

Correct Answer : A

Answer Justification :

Justification: Statement 1: It was indigenously developed by Defence Research and Development


Organisation (DRDO) under the Integrated Guided-Missile Development Programme (IGMDP).

Statement 2: It is powered by Ramjet-rocket propulsion system (RRPS) which renders thrust for the
missile to intercept the target at supersonic speed without any retardation.

Statement 3: It is capable of neutralising aerial targets like cruise missiles, fighter jets, unmanned
aerial vehicles (UAV) and air-to-surface missiles. It is a surface to air missile.

Q Source:Sometimes seen in news

73 The President places the report of which of the following bodies before the Parliament?
⌰〰〰
1. Union Public Service Commission
2. Comptroller and Auditor General of India
3. National Commission for STs
4. National Human Rights Commission

Select the correct answer using the codes below.


A. 1 and 2 only
B. 3 and 4 only
C. 1, 2 and 4 only
D. 1, 2, 3 and 4 only

Correct Answer : D

Answer Justification :

Learning: This is about the National Commission for STs.

The Commission presents an annual report to the President. It can also submit a report as and when
it thinks necessary.

The President places all such reports before the Parliament, along with a memorandum
explaining the action taken on the recommendations made by the Commission.
The memorandum should also contain the reasons for the non-acceptance of any of such

50
Total Marks : 200
Online Prelims TEST - 34 (TEXTBOOK)
( InsightsIAS Mock Test Series for UPSC Preliminary Exam 2020 ) Mark Scored : 0

recommendations.
The President also forwards any report of the Commission pertaining to a state government to
the state governor.
The governor places it before the state legislature, along with a memorandum explaining the
action taken on the recommendations of the Commission.

Q Source: Chapter 43: Indian Polity: M Laxmikanth

74 Which of the following is a component of the Gandhian way of Satyagaraha?


1. Living faith in God
2. Give up all material possessions
3. Fearlessness

Select the correct answer using the codes below.


A. 1 only
B. 2 and 3 only
C. 1 and 3 only
D. 1, 2 and 3

Correct Answer : C

Answer Justification :

Justification: Gandhi felt that Satyagraha was equally applicable to large-scale political struggle
and to one-on-one interpersonal conflicts and that it should be taught to everyone.

He listed seven rules as “essential for every Satyagrahi in India”:

must have a living faith in God


must believe in truth and non-violence and have faith in the inherent goodness of human
nature which he expects to evoke by suffering in the satyagraha effort
must be leading a chaste life, and be willing to die or lose all his possessions
must be a habitual khadi wearer and spinner
must abstain from alcohol and other intoxicants
must willingly carry out all the rules of discipline that are issued
must obey the jail rules unless they are specially devised to hurt his self-respect

Q Source: Chapter 26: India’s Struggle for Independence: Bipin Chandra

75 Which of the following causes a fatal blood disease, sometimes noted in the wool industry?

A. Anthrax
B. Clostridium
C. Aspergillus
D. Salmonella

51
Total Marks : 200
Online Prelims TEST - 34 (TEXTBOOK)
( InsightsIAS Mock Test Series for UPSC Preliminary Exam 2020 ) Mark Scored : 0

Correct Answer : A

Answer Justification :

Learning: Anthrax causes a fatal blood disease called sorter’s disease.

Option C:Aspergillus causes pulmonary and blood infections.

Option D: The well-known Typhoid fever is caused by Salmonella typhi.

Option B:Clostridium Botulinum toxin attacks the nervous system of the affected individual and
causes trouble in respiration, swallowing, speaking, vision and causes overall physical weakness.

Q Source: 6th Science NCERT

76 The Prayag Assembly convened by Harshavardhana is best known for being

A. An assembly of universal character for offerings of royal charities to all classes of people
B. An assembly for compilation of all major teachings of Buddhism
C. A meeting of all literary gems in Harsha’s Kingdom for showcasing the finest literature
produced during the reign of Harsha
D. A consortium of all merchants from major Kiingdoms to promote trade between Kingdoms

Correct Answer : A

Answer Justification :

Learning: In the year 643 A.D., Harsha held a great religious assembly in his capital at Kanauj on
the bank of the river Ganges. The purpose of the assembly was to highlight the teachings of
Buddha.

The Kanauj Assembly was followed by another spectacular assembly at Prayaga in the same year.

While the Kanauj Assembly was a religious assembly to highlight Mahayanism, the Prayaga
Assembly was an assembly of universal character for offerings of royal charities to all classes of
people.

It was known as the Maha Moksha Parishud. Harsha was at his best in the Prayaga Assembly as a
generous monarch and an admirer of all the major faiths of his country.

Q Source: Based on UPSC papers

77 The states which do NOT have an international border are


1. Haryana
2. Gujarat
3. Jharkhand

52
Total Marks : 200
Online Prelims TEST - 34 (TEXTBOOK)
( InsightsIAS Mock Test Series for UPSC Preliminary Exam 2020 ) Mark Scored : 0

4. Assam

Select the correct answer using the codes below.


A. 1, 3 and 4 only
B. 2 and 4 only
C. 1 and 3 only
D. 1, 2, 3 and 4

Correct Answer : C

Answer Justification :

Learning: Gujarat is bordered with Pakistan. But, it is also a coastal state.

Assam borders with Bangladesh and Bhutan, but is a landlocked state.

Chandigarh and Delhi are two other UTs which do not have a connection with neither international
borders nor the coasts.

Q Source: Revision previous tests syllabus: 9th Geography NCERT

78 What do you understand by ‘responsible government’ in the context of British India?

A. The Governor was required to act with the advice of ministers responsible to the provincial
legislature.
B. Government of India took major steps in promoting the welfare of vulnerable sections of
Indian population
C. The Viceroy, Governor and Ministers would conduct themselves strictly based on laws
D. Bureaucracy will act as per the instructions of the elected representatives instead of the
Governor and Viceroy

Correct Answer : A

Answer Justification :

Learning: Responsible government is a system which embodies the principle of parliamentary


accountability, such as in India.

It is also the foundation of the Westminster system of parliamentary democracy.

Government is responsible to the parliament rather than to the monarch, or, in a colonial context, to
the imperial government.

If the parliament is bicameral, then the government is responsible first to the parliament's lower
house, which is more numerous, directly elected and thus more representative than the upper
house.

53
Total Marks : 200
Online Prelims TEST - 34 (TEXTBOOK)
( InsightsIAS Mock Test Series for UPSC Preliminary Exam 2020 ) Mark Scored : 0

Q Source: Chapter 1: Indian Polity: M Laxmikanth

79 The right to property is a legal/constitutional right and not a fundamental right. This has which of the
following implications?
1. There is no guaranteed right to compensation in case of acquisition of a private property by the
state.
2. It can be regulated without a constitutional amendment by an ordinary lawof the Parliament.

Which of the above is/are correct?


A. 1 only
B. 2 only
C. Both 1 and 2
D. None

Correct Answer : C

Answer Justification :

Justification: Statement 1: For e.g. the earlier Land Acquisition Act did not provide for
resettlement and rehabilitation after the acquisition of private land by the state.

Statement 2: Since it is a legal right, it can be curtailed by the Parliament through an ordinary law.

Other implications are:

It protects private property against executive action but not against legislative action.
In case of violation, the aggrieved person cannot directly move the Supreme Court under
Article 32 (right to constitutional remedies including writs) for its enforcement. He can move
the High Court under Article 226.

Q Source: Present Position of Right to Property: Chapter 7: Indian Polity: M Laxmikanth

80 The southernmost, among the following, of all the states in the United States of America is

A. Texas
B. Michigan
C. Kansas
D. Oklahoma

Correct Answer : A

Answer Justification :

Learning: USA is the most affected country by COVID-19. The question was framed in that regard.

54
Total Marks : 200
Online Prelims TEST - 34 (TEXTBOOK)
( InsightsIAS Mock Test Series for UPSC Preliminary Exam 2020 ) Mark Scored : 0

Please refer to the map below.

Q Source: Map based questions

81 Tropical cyclones dissipate on reaching the land due to

A. Moisture supply cut off


B. Friction with local winds
C. Coriolis force
D. Change in air pressure

Correct Answer : A

Answer Justification :

Learning: Tropical cyclones originate and intensify over warm tropical oceans.

The energy that intensifies the storm comes from the condensation process in the towering
cumulonimbus clouds, surrounding the centre of the storm.

With continuous supply of moisture from the sea, the storm is further strengthened.

On reaching the land the moisture supply is cut off and the storm dissipates.

Q Source: 11th Fundamentals of Physical Geography NCERT

55
Total Marks : 200
Online Prelims TEST - 34 (TEXTBOOK)
( InsightsIAS Mock Test Series for UPSC Preliminary Exam 2020 ) Mark Scored : 0

82 Consider the following statements.


1. It is the largest protected area in the Eastern Himalaya biodiversity hotspot.
2. The park harbours the northernmost lowland evergreen rainforests in the world.
3. The habitat changes with increasing altitude from tropical moist forests to Alpine meadows and
perennial snow.
4. The park has extensive bamboo forests.

The above refer to?


A. Namdapha National Park
B. Balpakram National Park
C. Nongkhyllem Wildlife Sanctuary
D. Nokrek National Park

Correct Answer : A

Answer Justification :

Learning: It is located in Arunachal Pradesh in Northeast India. It is also the third largest national
park in India in terms of area. It is located in the Eastern Himalayan sub-region and is recognized
as one of the richest areas in biodiversity in India.

The park is located between the Dapha bum range of the Mishmi Hills and the Patkai range.

The habitat changes with increasing altitude from tropical moist forests to Montane forests,
temperate forests and at the higher elevations, to Alpine meadows and perennial snow. The park
has extensive bamboo forests and secondary forests in addition to the primary forests.

Q Source: Protected areas

83 The Constitution says that these principles are “fundamental in the governance of the country”:

A. Preamble
B. Directive Principles of State Policy
C. Fundamental Rights
D. Fundamental Duties

Correct Answer : B

Answer Justification :

Justification : Article 37 says that these principles are fundamental in the governance of the
country and it shall be the duty of the State to apply these principles in making laws.

Options (a) talks about ideal values. Option (d) is about the code of conduct expected of a citizen.
Option (c) are values in action.

56
Total Marks : 200
Online Prelims TEST - 34 (TEXTBOOK)
( InsightsIAS Mock Test Series for UPSC Preliminary Exam 2020 ) Mark Scored : 0

Q Source: Basics: Polity

84 Oceanic crust

A. Is formed by asteroid impact.


B. Is enriched in iron and magnesium compared to continental crust.
C. Forms from sea salt.
D. Is made of metamorphic rock.

Correct Answer : B

Answer Justification :

Learning: The crust overlies the solidified and uppermost layer of the mantle.

Oceanic crust is the result of erupted mantle material originating from below the plate, cooled and
in most instances, modified chemically by seawater.

It is primarily composed of mafic rocks, or sima, which is rich in iron and magnesium.

It is thinner than continental crust, or sial, generally less than 10 kilometers thick; however it is
denser

Q Source: Chapter 2: Goh Cheng Leong - Certificate Physical and Human Geography

85 Which of the following aligns most closely with the views of Gandhi on mass production or
industrialization?

A. He feared that mass production would lead to greater exploitation of class by class and
urbanization.
B. He believed that post-independent India did not have the resources to go for large-scale
production.
C. He did not see the basic necessities of life being fulfilled by mass production.
D. He believed that mass production would again lead to enslavement of India by foreigners.

Correct Answer : A

Answer Justification :

Justification: Gandhian economics places importance to means of achieving the aim of


development and this means must be non-violent, ethical and truthful in all economic spheres.

In order to achieve this means he advocated trusteeship, decentralization of economic


activities, labour-intensive technology and priority to weaker sections.
The revival of the economy is made possible only when it is free from exploitation, so

57
Total Marks : 200
Online Prelims TEST - 34 (TEXTBOOK)
( InsightsIAS Mock Test Series for UPSC Preliminary Exam 2020 ) Mark Scored : 0

according to Gandhi, industrialization on a mass-scale will lead to passive or active


exploitation of the people as the problem of competition and marketing comes in.
Gandhi believes that for an economy to be self-contained, it should manufacture mainly for its
use even if that necessitates the use of modern machines and tools, provided it is not used as
a means of exploitation of others.

Q Source: Improvisation: Chapter 24: India’s Struggle for Independence: Bipin Chandra

86 Rashtriya Bal SwasthyaKaryakram (RBSK) is an important initiative aiming at

A. Providing medical facilities for adolescents in every primary school


B. Post-birth immediate insurance coverage of all health related problems
C. Early identification and early intervention for children from birth to adulthood for
developmental delays and birth defects
D. Free provision of nutritional supplements and essential medicines for new born children

Correct Answer : C

Answer Justification :

Learning: According to March of Dimes (2006), out of every 100 babies born in this country
annually, 6 to 7 have a birth defect. This would translate to around 17 lakhs birth defects annually
in the country and accounts for 9.6% of all the newborn deaths.

So, RBSK aims at early identification and early intervention for children from birth to 18 years to
cover 4 ‘D’s viz. Defects at birth, Deficiencies, Diseases, Development delays including disability.

Early detection and management diseases including deficiencies bring added value in preventing
these conditions to progress to its more severe and debilitating form and thereby reducing
hospitalization and improving implementation of Right to Education.

Q Source: Chapter 16: IYB 2020

87 The Congress proposed legislation for the reduction of rent and the restoration of Bakasht lands.
Consider the following about it.
1. The Bakasht landissue became the reason for the merger of the Bihar Provincial Kisan Sabha with
the Indian National Congress (INC).
2. The Bakasht land movement continued untillthe non-cooperation movement came to a halt.

Which of the above is/are true?


A. 1 only
B. 2 only
C. Both 1 and 2
D. None of the above

58
Total Marks : 200
Online Prelims TEST - 34 (TEXTBOOK)
( InsightsIAS Mock Test Series for UPSC Preliminary Exam 2020 ) Mark Scored : 0

Correct Answer : D

Answer Justification :

Learning: The Bakasht lands issue became a major ground of contention between the Bihar
Provincial Kisan Sabha and the Congress Ministry. The Bakasht land movement continued till 1945
till zamindari was abolished.

Bakasht lands were those which the occupancy tenants had lost to zamindars, mostly during the
Depression years, by virtue of nonpayment of rent, and which they often continued to cultivate as
share-croppers.

But the formula that was finally incorporated in the legislation on the basis of an agreement with
the zamindars did not satisfy the radical leaders of the kisan Sabha.

The legislation gave a certain proportion of the lands back to the tenants on condition that they pay
half the auction price of the land. Besides, certain categories of land had been exempted from the
operation of the law.

Q Source: Chapter 27: India’s Struggle for Independence: Bipin Chandra

88 The competent authority to change the name of a State of India is the

A. State Legislature concerned


B. Governor concerned
C. Parliament of India
D. Union Home Minister

Correct Answer : C

Answer Justification :

Learning: The Constitution authorizes the Parliament to form new states or alter the areas,
boundaries or names of the existing states without their consent.

For e.g. the United Provinces was the first state to have a new name. It was renamed ‘Uttar
Pradesh’ in 1950. In 1969, Madras was renamed ‘Tamil Nadu’.

Q Source: Chapter 5: Indian Polity: M Laxmikanth

89 'Oort cloud’ around our solar system are believed to be thick bubbles of

A. Cosmic thunderstorms
B. Electromagnetic radiation
C. Icy debris
D. Water vapor around gaseous planets

59
Total Marks : 200
Online Prelims TEST - 34 (TEXTBOOK)
( InsightsIAS Mock Test Series for UPSC Preliminary Exam 2020 ) Mark Scored : 0

Correct Answer : C

Answer Justification :

Learning:About ‘solar system’: one of the first things you will hear (apart from planets) is Kuiper
Belt and Oort clouds.

The Oort Cloud is believed to be a thick bubble of icy debris that surrounds our solar system. This
distant cloud may extend a third of the way from our sun to the next star -- between 5,000 and
100,000 astronomical units. Earth is about one astronomical unit from the sun (roughly 93 million
miles or 150 million kilometers).

Dutch astronomer Jan Oort first proposed the idea of this region of space to explain the origins of
comets with that take thousands of years to orbit the sun.

Here http://solarsystem.nasa.gov/planets/

Q Source: AR:Chapter 1: Goh Cheng Leong - Certificate Physical and Human Geography

90 Which of the following most appropriately describes INS Arihant?

A. It is India’s largest aircraft carrier.


B. It is India’s first indigenously built nuclear powered submarine that can also fire nuclear
weapons.
C. It is Indian Navy’s first ground borne surveillance system.
D. None of the above

Correct Answer : B

Answer Justification :

Learning: It is a 6,000-tonne nuclear powered and armed submarine and is more than 100 metre
long. The launch of Arihant strengthens India's endeavour to build a credible nuclear triad – the
capability to fire nuclear weapons from air, land and sea.

It is powered by a pressurised light water reactor built with Russia’s help. This reactor generates
tremendous heat, driving a steam turbine

Presently, Indian Navy operates the INS Chakra, a nuclear-powered submarine which is leased for
10 years from Russia in 2012.

Q Source: Often in news

91 The Roaring Forties are caused due to


1. Air being displaced from the Equator towards the South Pole
2. Earth’s rotation

60
Total Marks : 200
Online Prelims TEST - 34 (TEXTBOOK)
( InsightsIAS Mock Test Series for UPSC Preliminary Exam 2020 ) Mark Scored : 0

3. Equatorial counter-currents
4. Thermal dipole created in the Pacific Ocean

Select the correct answer using the codes below.


A. 1, 3 and 4 only
B. 2 and 3 only
C. 1 and 2 only
D. 1, 2, 3 and 4

Correct Answer : C

Answer Justification :

Learning: Theseare strong westerly winds found in the Southern Hemisphere.

Hot air rises at the Equator and is pushed towards the poles by cooler air travelling towards the
Equator (an atmospheric circulation feature known as the Hadley Cell).

At about 30 degrees from the equator, the outward-travelling air sinks to lower altitudes, and
continues toward the poles closer to the ground (the Ferrel Cell), then rises up again from about 60
degrees as the air joins the Polar vortex.

This travel in the 30 to 60 degree zone combines with the rotation of the earth to move the air
currents from west to east, creating westerly winds.

Q Source: Based on UPSC papers

92 Consider the following statements.


1. Most phosphorus in nature exists in the form of phosphate ion.
2. Phosphorous is often the most abundant nutrient, i.e.the nutrient that is most available and thus
promotes growth, in aquatic ecosystems.
3. When nitrogen and phosphorus from fertilizer are carried in runoff to lakes and oceans, they can
cause eutrophication.

Select the correct answer using the codes below.


A. 2 and 3 only
B. 1, 2 and 3
C. 1 and 3 only
D. 1 only

Correct Answer : C

Answer Justification :

Justification: Phosphorus is an essential nutrient found in the macromolecules of humans and


other organisms, including DNA.

61
Total Marks : 200
Online Prelims TEST - 34 (TEXTBOOK)
( InsightsIAS Mock Test Series for UPSC Preliminary Exam 2020 ) Mark Scored : 0

The phosphorus cycle is slow. Most phosphorus in nature exists in the form of phosphate ion—PO43-.

Phosphorus is often the limiting nutrient, or nutrient that is most scarce and thus limits growth, in
aquatic ecosystems.

When nitrogen and phosphorus from fertilizer are carried in runoff to lakes and oceans, they can
cause eutrophication, the overgrowth of algae. The algae may deplete oxygen from the water and
create a dead zone.

93 The Table of Precedence is related to the rank and order of the officials of the Union and State
Governments. Who among the following is ranked the lowest in this table?

A. Cabinet Ministers of the Union


B. Speaker of Lok Sabha
C. Former Presidents
D. Governors of states within their respective states

Correct Answer : A

Answer Justification :

Learning:The table goes like this:

I. President
II. Vice-President
III. Prime Minister
IV. Governors of states within their respective states
V. Former presidents; Deputy Prime Minister
VI. Chief Justice of India; Speaker of Lok Sabha
VII. Cabinet Ministers of the Union; Chief Ministers of States within their respective States;
Deputy Chairman, Planning Commission; Former Prime Ministers; Leaders of Opposition in
Rajya Sabha and Lok Sabha

Q Source: Appendices: Indian Polity: M Laxmikanth

94 Slate is related to shalein the same way that

A. Gneiss is related to marble.


B. Marble is related to limestone
C. Gravel is related to siltstone.
D. Basalt is related to granite.

Correct Answer : B

62
Total Marks : 200
Online Prelims TEST - 34 (TEXTBOOK)
( InsightsIAS Mock Test Series for UPSC Preliminary Exam 2020 ) Mark Scored : 0

Answer Justification :

Learning: When a metamorphic rock is formed under pressure, its crystals become arranged in
layers. Slate, which is formed from shale, is like this.

Metamorphic rocks sometimes contain fossils if they were formed from a sedimentary rock, but the
fossils are usually squashed out of shape.

The main difference between limestone and marble is that limestone is a sedimentary rock, typically
composed of calcium carbonate fossils, and marble is a metamorphic rock.Marble forms when
sedimentary limestone is heated and squeezed by natural rock-forming processes so that the grains
recrystallize.

Q Source: Chapter 1: Goh Cheng Leong - Certificate Physical and Human Geography

95 Which of the following is NOT an ionizing radiation?

A. X-rays
B. Gamma rays
C. Microwaves
D. Cosmic rays

Correct Answer : C

Answer Justification :

Learning: Such radiation carries enough energy to free electrons from atoms or molecules, thereby
ionizing them.

Gamma rays, X-rays, and the higher ultraviolet part of the electromagnetic spectrum are ionizing,
whereas the lower ultraviolet part of the electromagnetic spectrum, and also the lower part of the
spectrum below UV, including visible light (including nearly all types of laser light), infrared,
microwaves, and radio waves are all considered non-ionizing radiation.

Ionizing radiation can also be generated artificially using X-ray tubes, particle accelerators, and any
of the various methods that produce radioisotopes artificially.

Ionizing radiation is invisible and not directly detectable by human senses

Q Source: 9th Standard ICSE Textbook: Unit II

96 Consider the following statements.


1. Mycoremediation is a form of bioremediation in which fungi are used to decontaminate an area.
2. Wood-degrading fungi can be used to breakdown aromatic pollutants such as toxic components of
petroleum in the event of an oil spill.

Select the correct answer using the codes below.

63
Total Marks : 200
Online Prelims TEST - 34 (TEXTBOOK)
( InsightsIAS Mock Test Series for UPSC Preliminary Exam 2020 ) Mark Scored : 0

A. 1 only
B. 2 only
C. Both 1 and 2
D. None of the above

Correct Answer : C

Answer Justification :

Justification: Bacteria are widely diverse organisms, and thus make excellent players in
biodegradation and bioremediation.

There are few universal toxins to bacteria, so there is likely an organism able to break down any
given substrate, when provided with the right conditions (anaerobic versus aerobic environment
etc).

Statement 1:Mycoremediation is a form of bioremediation in which fungi are used to decontaminate


the area.

The toxins can include heavy metals, persistent organic pollutants, textile dyes, leather tanning
industry chemicals and wastewater, petroleum fuels, polycyclic aromatic hydrocarbon,
pharmaceuticals and personal care products, pesticides and herbicide,in land, fresh water and
marine environments. The byproducts of the remediation can be valuable materials themselves,
such as enzymes (like laccase), edible or medicinal mushrooms,making the remediation process
even more profitable.

Statement 2: Wood-degrading fungi (an example of mycoremediation) are particularly effective in


breaking down aromatic pollutants (toxic components of petroleum), as well as chlorinated
compounds (certain persistent pesticides).

Q Source: 9th Standard ICSE Textbook: Unit II

97 Financial Stability and Development Council (FSDC) is an autonomous body dealing with macro
prudential and financial regularities in the entire financial sector of India. Its members include
1. Governor Reserve Bank of India (RBl)
2. Representative, Financial Action Task Force (FATF)
3. Chief Economic Advisor, Ministry of Finance
4. Chairman, Securities and Exchange Board of India (SEBI)

Select the correct answer using the codes below.


A. 1 and 3 only
B. 1, 3 and 4 only
C. 2 and 4 only
D. 1, 2, 3 and 4

Correct Answer : B

64
Total Marks : 200
Online Prelims TEST - 34 (TEXTBOOK)
( InsightsIAS Mock Test Series for UPSC Preliminary Exam 2020 ) Mark Scored : 0

Answer Justification :

Learning:It is an apex-level body constituted by the government of India.

Chairperson: The Union Finance Minister of India

Members:

Governor Reserve Bank of India (RBl),


Finance Secretary and/ or Secretary, Department of Economic Affairs (DEA),
Secretary, Department of Financial Services (DFS),
Secretary, Ministry of Corporate Affairs,
Secretary, Ministry of Electronics and Information Technology,
Chief Economic Advisor, Ministry of Finance,
Chairman, Securities and Exchange Board of India (SEBI),
Chairman, Insurance Regulatory and Development Authority (IRDA),
Chairman, Pension Fund Regulatory and Development Authority (PFRDA),
Chairman, Insolvency and Bankruptcy Board of India (IBBI),

Additional Secretary, Ministry of Finance, DEA, is the Secretary of the Council.

Responsibilities

Financial Stability
Financial Sector Development
Inter-Regulatory Coordination
Financial Literacy
Financial Inclusion
Macro prudential supervision of the economy including the functioning of large financial
conglomerates
Coordinating India's international interface with financial sector bodies like the Financial
Action Task Force (FATF), Financial Stability Board (FSB)and any such body as may be
decided by the Finance Minister from time to time.

Q Source: Ministry of Finance Website

98 The Governor cannot promulgate an ordinance, at all, without the instructions from the President in
which of the following cases?
1. If the same act of the state legislature would have been invalid without receiving the President’s
assent
2. If the subject of ordinance falls in the concurrent list

Which of the above is/are correct?


A. 1 only
B. 2 only
C. Both 1 and 2
D. None

65
Total Marks : 200
Online Prelims TEST - 34 (TEXTBOOK)
( InsightsIAS Mock Test Series for UPSC Preliminary Exam 2020 ) Mark Scored : 0

Correct Answer : A

Answer Justification :

Justification: Statement 1:He needs to take President’s instructions if a bill containing the same
provisions would have required the previous sanction of the President for its introduction into the
state legislature.

Statement 1:Only if a Central law exists on the matter in the concurrent list, the Governor may
require the prior instruction of the President. Otherwise, it is fine and he can go ahead without
President’s consent.

Q Source: Chapter on Governor: Indian Polity: M laxmikanth

99 Consider the following matches of Valley with states/UTs.


1. Markha Valley : Ladakh
2. Dzukou Valley: Himachal Pradesh
3. Sangla Valley: Assam
4. Yumthang Valley: Sikkim

Select the correct answer using the codes below.


A. 3 and 4 only
B. 1 and 2 only
C. 2 and 3 only
D. 1 and 4 only

Correct Answer : D

Answer Justification :

Justification: The Markha River is a river in Ladakh, India. It is a tributary of the Zanskar
River.The Markha Valley is one of the most popular trekking routes in Ladakh,accessible from
Ganda La pass near Spituk in the west, which is usually the beginning point of the trek,and
Gongmaru La pass near Hemis,where the trek usually ends.

The Dzukou Valley is a valley located at the border of the Indian states of Nagaland and Manipur.

Sangla valley is in the Kinnaur District of Himachal Pradesh close to the Tibetan border.

The Yumthang Valley is a grazing pasture surrounded by the Himalayan mountains in the North
Sikkim.

Q Source: Past year UPSC questions

100 Which of the following are set up under an Act of the Parliament?
1. University Grants Commission

66
Total Marks : 200
Online Prelims TEST - 34 (TEXTBOOK)
( InsightsIAS Mock Test Series for UPSC Preliminary Exam 2020 ) Mark Scored : 0

2. National Green Tribunal


3. National Consumer Disputes Redressal Commission

Select the correct answer using the codes below.


A. 1 and 2 only
B. 1 and 3 only
C. 2 and 3 only
D. 1, 2 and 3

Correct Answer : D

Answer Justification :

Justification: The University Grants Commission (UGC) of India is a statutory body set up by the
Indian Union government in accordance to the UGC Act 1956.

The National Green Tribunal was established under the National Green Tribunal Act 2010 for
effective and expeditious disposal of cases relating to environmental protection and conservation of
forests and other natural resources.

The National Consumer Disputes Redressal Commission (NCDRC), India is a quasi-judicial


commission in India which was set up in 1988 under the Consumer Protection Act of 1986.

Q Source: Unit on Statutory bodies: Indian Polity: M Laxmikanth

67

You might also like